Você está na página 1de 53

Egyptian_Pediatric yahoo group

Egyptian_Pediatric yahoo group

http://health.groups.yahoo.com/group/ egyptian_pediatric/

http://health.groups.yahoo.com/group/ egyptian_pediatric/

SELF-ASSESSMENT

Self-assessment short cases


Questions
Question 1 A 2-year-old girl presented to her district general hospital with a history of cough, high fever and lethargy for 5 days while on holiday in Spain. She had a past history of recurrent infections, including staphylococcal meningitis and severe chickenpox requiring intravenous aciclovir, and had been investigated for neutropenia, which had resolved spontaneously and was thought to be transient neutropenia of childhood. For the previous 12 months, she had remained very well. On examination, she was well grown but was pale and had numerous fresh bruises on her limbs and trunk. She had a 4 cm smooth palpable liver and massive splenomegaly but no lymphadenopathy. Her chest was clear, there were no signs of infection, and examination was otherwise normal. A blood test showed:
Haemoglobin White cell count Platelet count RBC MCV PCV MCH Neutrophil count Lymphocyte count Monocyte count Eosinophil count 6.0 g/dl 3.52 109/l 50 109/l 3.07 1012/l 66.8 0.205 19.5 pg 0.56 109/l 2.65 109/l 0.29 109/l 0.01 109/l

well and had dry nappies so was taken to the emergency department by her mother. Mum said that the baby had had a distended abdomen since birth but that it had become increasingly distended, especially on the left side. The baby had not been eating well and had lost some weight over the previous 2 months. On examination, she was noted to be pale. She had no dysmorphic features. She had a temperature of 37.6 1C, a mild tachycardia and a blood pressure of 147/ 97 mmHg. On palpation, a left-sided abdominal mass was evident, and a CT scan showed an intrarenal mass with calcication and a single pulmonary nodule. The results of the investigations were as follows:
Haemoglobin White cell count Platelet count Urea and electrolytes Liver function tests Prothrombin time APTT D-dimers Fibrinogen Urine noradrenalin Urine dopamine: DOP Urine HMMA Urine HVA 10.2 g/dl 31 109/l (neutrophilia) 899 109/l Normal Normal Normal Prolonged Normal Normal Normal Normal Normal Normal

Blood lm: Pancytopenia and hypochromic anaemia with microcytosis. A small number of possible lymphoblasts and occasional nucleated red blood cells. Bone marrow aspiration was performed (Figure 1). 1. What is the diagnosis? a. Acute leukaemia b. Leishmaniasis c. Lymphoma d. Chronic leukaemia e. Metabolic storage disorder 2. What Is the recommended treatment? 3. In this case, what further investigations would be warranted? Question 2 A 14-month-old child presented with a 2-week history of diarrhoea and vomiting but no fever. She had not been drinking
Jenny Adamski MBBS Paediatric Specialist Registrar at the Royal Manchester Childrens Hospital, Pendlebury, Manchester, UK. Stephen D Playfor MD Consultant Paediatric Intensivist and Honorary Clinical Lecturer in Paediatric Intensive Care Medicine at the Paediatric Intensive Care Unit, Royal Manchester Childrens Hospital, Pendlebury, Manchester, UK.

1. What is the most likely diagnosis? a. Neuroblastoma b. Lymphoma c. Wilms tumour d. Clear cell sarcoma of the kidney e. Mesoblastic nephroma 2. What is the cause of her clotting abnormality? a. Haemophilia b. Disseminated intravascular coagulation (DIC) c. Von Willebrands disease d. A heparin-contaminated sample e. Liver disease 3. Which is the most appropriate treatment for her hypertension? a. Observation only b. Fluid restriction and diuretics c. Angiotensin-converting enzyme (ACE) Inhibitor d. Sodium restriction e. b-Blocker Question 3 A 3-year-old Afro-Caribbean boy presented with a history of having been unwell for 2 days with pyrexia, vomiting and lethargy. Mum commented that he had not been drinking well, and she felt he was a little dehydrated. He had not been eating well either, although he was a very picky eater and this was not unusual for him. His urine had been red and had now become very dark like Coca-Cola. On examination, he was pyrexial with a temperature of 39.1 1C, although his extremities were

PAEDIATRICS AND CHILD HEALTH 17:1

31

r 2007 Elsevier Ltd. All rights reserved.

SELF-ASSESSMENT

weeks his cough had become worse and his inhaler did not seem to be working. His breathing was particularly noisy at night, and he had become progressively short of breath, struggling to climb the stairs. In himself, he had remained active, and his temperature had settled. His mum said he had not been eating well during this time and may have lost some weight. On examination, he was well grown and comfortable at rest with no respiratory distress. His chest revealed quiet breath sounds bilaterally and scattered wheeze. Examination was otherwise unremarkable other than a small, 2 cm, mobile lymph node in the right supraclavicular region. 1. Which would be the most useful initial investigation to make a diagnosis? a. Lung function tests b. Chest X-ray c. Full blood count, C-reactive protein (CRP) level and sputum culture d. Sweat test The following investigations were carried out:
Hb WCC Platelets CRP LDH CXR (Figure 2) 11.2 15.22 346 83 1074

Figure 1 Bone marrow aspirate.

cold and he had a mild cyanotic appearance to his ngers and toes. He was noted to be very pale and mildly jaundiced, although there was no rash and examination was otherwise normal. Investigations revealed the following results:
Hb MCV WBC Platelets Reticulocytes ZPP Direct Coombs test +++++ 1.7 g/dl (1214 g/dl) 61 (7388 ) 22.75 109/l (5.015.0 109/l) 352 109/l (150400 109/l) 0.46% (26%) 4600

1. What is the most likely diagnosis? a. Hereditary spherocytosis b. Haemolytic uraemic syndrome c. Malaria d. Paroxysmal cold haemoglobinuria e. Warm autoimmune haemolytic anaemia 2. What test would you do to conrm your diagnosis? 3. What is the single most important therapeutic intervention? 4. What other pathology accounts for the severity of the anaemia?

2. What three further biochemical investigations would you do? 3. What is the most likely diagnosis? a. Lobar pneumonia b. Asthma c. Lymphoma d. Pulmonary tuberculosis e. Cardiac failure

Question 4 A 3-year-old boy presented with a 4-week history of cough, intermittent temperature and shortness of breath. On presentation to the GP 3 weeks previously, he had been coryzal, and examination had revealed bilateral wheeze. He was also known to have a strong family history of asthma in his mum and older sister, and had frequently been chesty as a baby. The GP diagnosed him as having asthma and treated him with a salbutamol inhaler, a 3-day course of oral steroids and oral antibiotics. His mum reported that this had made a big difference and he had been well for a week, but in the past 2

Figure 2 Chest radiograph.

PAEDIATRICS AND CHILD HEALTH 17:1

32

r 2007 Elsevier Ltd. All rights reserved.

SELF-ASSESSMENT

Figure 3 Radiograph left knee (a); radiograph right knee (b); chest radiograph (c).

4. On admission to hospital, he was noted to have stridor at night. What would the next most appropriate course of action be? a. Intravenous antibiotics b. Oral or intravenous steroids c. Urgent biopsy under general anaesthesia d. Nebulised adrenaline e. Bronchoscopy

Question 5 A 12-year-old boy was admitted to the emergency department with difculty breathing. He had been diagnosed with Duchenne muscular dystrophy at 6 years of age and had been wheelchair-bound since 8 years old. He had a mild scoliosis with good cardiac and respiratory function, and had never been mechanically ventilated. In the emergency department, he was

pyrexial to 39.7 1C and agitated, with oxygen saturations of 50% despite being given 100% oxygen by facemask. His respiratory rate was 60 breaths/min, his heart rate 165 beats/min and his blood pressure 95/58 mmHg. The boy had been slowly recovering from an upper respiratory tract infection over the preceding 2 weeks and had slipped forwards out of his wheelchair onto his knees 30 hours before presentation. Endotracheal intubation was immediately carried out, and oxygenation proved difcult. X-rays of both knees and the chest were performed (Figure 3ac).

1. What is the most likely diagnosis? a. Bronchopneumonia b. Pulmonary haemorrhage c. Fat embolism syndrome

PAEDIATRICS AND CHILD HEALTH 17:1

33

r 2007 Elsevier Ltd. All rights reserved.

SELF-ASSESSMENT

d. Pulmonary oedema e. Acute severe asthma 2. What are the usual clinical features of this condition? 3. What specic treatment is indicated?

97% patients.3 Leishmaniasis is more common in people with immunodeciency, and in view of this childs past history of severe infection, investigations for immunodeciency would be warranted. Answer 2

Answers
Answer 1 1. bLeishmaniasis 2. Liposomal amphotericin 3. Investigations for immunodeciency Leishmaniasis is caused by protozoa of the Leishmania species, which are transmitted via the bite of a female sandy; it is clinically classied into cutaneous, mucocutaneous and visceral Leishmaniasis. Visceral leishmaniasis is most common in Bangladesh (where it is known as kala azar), Brazil, India, Nepal and Sudan, but it is also found in some parts of southern Europe. Visceral leishmaniasis in the Mediterranean basin is generally caused by Leishmania infantum, which mainly affects children aged 14 years. Also known as black disease owing to the characteristic darkening of the skin, it can range from being asymptomatic to a fulminant disease. Presentation is often with a fever, which can be continuous, intermittent or irregular, hepatosplenomegaly (secondary to the compensatory production of phagocytic blood cells), leukopenia, lymphadenopathy, weight loss, wasting, weakness and diarrhoea. In visceral leishmaniasis, patients may die of haemorrhage (secondary to inltration of the haematopoietic system), severe anaemia or secondary infections particularly of the mucous membranes, bacterial pneumonia, septicaemia, tuberculosis, dysentery or measles. The differential diagnosis is varied and includes other tropical causes such as malaria, tropical splenomegaly syndrome, schistosomiasis, African trypanosomiasis, miliary tuberculosis, brucellosis and typhoid fever, as well as other causes of massive hepatosplenomegaly such as portal hypertension, histoplasmosis, lymphoma and systemic lupus erythematosus. A common presentation is with pancytopenia and hepatomegaly, and it is thus often confused with leukaemia. The denitive diagnosis is made by observing the parasite (more specically, amastigotes in tissue) on stained Giemsa smears or by observing the culture of bone marrow or splenic, hepatic or lymph node aspirates. The bone marrow aspirate (Figure 1 above) shows macrophages containing amastigotes (the resting intracellular stage of leishmaniae, formerly known as LeishmanDonovan bodies, which are shown as small, dark blue dots within the macrophages in Figure 1). Although splenic aspirates are the most sensitive test to provide an accurate diagnosis, the technique is traumatic and invasive so is rarely performed. Mortality in untreated cases is high, but with early treatment, the cure rate is higher than 90%.1,2 The treatment of choice in children in Europe Is liposomal amphotericin, and although numerous dosing regimens have been used, a dose of 3 mg/kg on days 15 and a single dose of 3 mg/kg on day 10 seems optimal and achieves cure in over

1. cWilms tumour 2. cVon Willebrands disease 3. cACE inhibitors The most common cause of an abdominal mass in a 14month-old child is a neuroblastoma, followed by a Wilms tumour, and the most common cause of an intrarenal tumour is a Wilms tumour. Neuroblastomas are malignant tumours derived from the sympathetic nervous system and generally occur in the adrenal gland, although they can be intrarenal. Urinary catecholamines are raised in 90% of neuroblastomas, with only 510% of tumours not producing catecholamines; normal urinary catecholamine levels therefore make this an unlikely diagnosis. Mesoblastic nephroma Is the most common form of congenital renal tumour and is very rare after the age of 3 years. It is generally cured by surgical resection only, and metastases are extremely rare, excluding it from the differential diagnosis. Clear cell sarcomas of the kidney are rare and make up approximately 6% of renal tumours. They are aggressive tumours that are otherwise known as bone-metastasising tumours of the kidneys due to their propensity for bony metastasis; a bone scan is essential in these cases. Differentiation from a Wilms tumour is only possible on biopsy, but the rarity of the tumour stops it being the most likely diagnosis. Likewise, lymphoma of the kidney is rare. Wilms tumours are the most common genitourinary malignancy of childhood, affecting 1 in 10 000 children, 75% of children being under the age of 4 and 90% under the age of 7. Clinical presentation is generally with a well child, and 74% of children present with the nding of an incidental mass. Fever is unusual, occurring in 1% of patients, and pain occurs in 44%. Wilms tumours are associated with a number of genetic syndromes such as WAGR syndrome (Wilms tumour, aniridia, genitourinary malformations and mental retardation), DenysDrash syndrome (pseudohermaphroditism, nephrotic syndrome and Wilms tumour) and BeckwithWiedemann syndrome (exomphalos, macroglossia, neonatal hypoglycaemia, visceromegaly, hemihypertrophy, characteristic facial features and linear indentations of the ear lobe). It is therefore important to look for dysmorphic features. The most common site of metastasis is to the lungs.4 A rare but important complication of Wilms tumour is the development of acquired von Willebrands disease, which occurs in up to 8% of those with Wilms tumour.5 Acquired von Willebrands disease occurs in a number of different conditions, for example lymphoproliferative disorders, myeloproliferative syndromes, hypothyroidism, autoimmune diseases such as systemic lupus erythematosus, drugs, congenital heart valve defects, infectious diseases and rarely solid tumours. The mechanism behind acquired von Willebrands disease in Wilms tumour is felt not to be the production of von

PAEDIATRICS AND CHILD HEALTH 17:1

34

r 2007 Elsevier Ltd. All rights reserved.

SELF-ASSESSMENT

Willebrands disease inhibitors or the absorption of factor VIII or von Willebrand factor onto nephroblastoma cells, but to the production of hyaluronic acid by nephroblastoma cells; a plasma factor that interacts with von Willebrand factor and causes decreased serum factor levels, produced by nephroblastoma cells. The bleeding caused by acquired von Willebrands disease is generally mild to moderate and mirrors that seen in mild-to-moderate type 1 or 2 von Willebrands disease. In severe bleeding, it is controlled with cryoprecipitate or factor VIII concentrate. Acquired von Willebrands disease in Wilms tumour can be particularly refractory to treatment until the tumour has been treated with chemotherapy or surgery, after which it resolves. Hypertension in Wilms tumour is caused by a number of factors, including excess renin production, vascular compression and pre-existing renal disease. More recently, the importance of renin production by Wilms tumour has been implicated in associated hypertension. A recent study found that 81% of a cohort of 47 hypertensive patients had increased renin production.6 Increased renin production leads to an increased conversion of angiotensinogen to angiotensin I and subsequent conversion to angiotensin II by ACE, resulting in vasoconstriction and renal hypertension. The administration of an ACE inhibitor in this situation would therefore be an appropriate rstline treatment for Wilms-induced hypertension. As not all hypertension in renal tumours is caused by excess renin, other methods of blood pressure control may be necessary, although most tumour-induced hypertension will require treatment to minimise the long-term effects and risks during surgery. Hypertension resolves following surgical resection. Answer 3 1. 2. 3. 4. dParoxysmal cold haemoglobinuria DonathLandsteiner test or test for cold agglutinins Warming the patient peripherally Iron-deciency anaemia

The presenting features are that of a haemolytic anaemia with jaundice and anaemia. Hereditary spherocytosis usually causes a mild anaemia but can be more severe during infection, particularly with parvovirus infection, and malaria can cause intravascular haemolysis and dark urine known as blackwater fever. Haemolytic uraemia syndrome generally follows bloody diarrhoea caused by infection with Escherichia coli O157 and is a cause of microangiopathic haemolysis, in which activation of the clotting cascade causes the formation of intravascular brin meshes that cause physical destruction of the red blood cells. In all of these cases, the direct Coombs test is negative. Autoimmune haemolytic anaemias are caused by antibody production by the body against its own red cells and can be divided into warm and cold depending whether the antibody reacts more strongly with red cells at 37 1C or at 4 1C. Cold agglutins are immunoglobulins that are directed against antigens on the red cell surface and agglutinate red cells at cold temperatures, below the physiological temperature of 37 1C. Paroxysmal cold haemoglobinuria is a rare type of autoimmune haemolytic anaemia, occurring primarily in

children, caused by IgG antibodies (DonathLandsteiner antibodies) against the P antigen on the red blood cell membrane. Paroxysmal cold haemoglobinuria can be an after-effect of an infection, when a microorganism triggers the formation of antibodies. It is characterised by acute-onset severe anaemia and haemoglobinuria following cold exposure. The antibody binds with the red blood cells in the cold extremities, and then, as the blood returns to the warmer central circulation, lysis occurs the so-called biphasic response. It is often found associated with infections, particularly viral infections including measles, mumps, inuenza, adenovirus, chickenpox, cytomegalovirus, and EpsteinBarr virus, as well as bacterial infections, especially syphilis, Haemophilus inuenzae and Mycoplasma pneumoniae. In addition, it has been found in association with lymphoma and acute myeloid leukaemia. In the parts of the body that remain cool, such as the ngers, toes or tip of the nose, microvascular obstruction leads to the appearance of cyanosis and may evolve to gangrene if complete. Cold agglutinaemia can be transient or chronic but is more likely to be transient in children and resolves spontaneously. In warm autoimmune haemolysis, the antibodies are IgG rather than IgM, and intravascular haemolysis is unusual as destruction of the red blood cells occurs in the spleen and liver, and therefore haemoglobinuria is not generally a feature. In warm autoimmune haemolytic anaemias a high reticulocyte count is evident, whereas in paroxysmal cold haemoglobinuria (PCH) the reticulocyte count is low as the reticulocytes as preferentially destroyed. Diagnosis is conrmed by the presence of cold agglutins at 4 1C. The DonathLandsteiner test is performed by combining the patients serum and red blood cells at 4 1C, allowing complement to bind, warming to 37 1C and assessing whether haemolysis and the biphasic response have taken place. The most important therapeutic intervention is to warm the child and stop the agglutination; this is generally done in a side room to an ambient temperature of 3739 1C. Once haemolysis has been switched off, treatment is supportive with blood transfusion if required and treatment of the underlying infection. Although acute symptomatic haemolysis is common, severe anaemia to this extent is unusual, suggesting an underlying second pathology or chronic anaemia. In this case, the raised ZPP and history of a poor diet suggest irondeciency anaemia, and the child was treated with iron and folic acid on recovery. Answer 4 1. b Chest X-ray 2. Urinary catecholamines, alphafetoprotein and b human chorionic gonadotropin 3. c Lymphoma 4. b Steroids The chest X-ray shown in Figure 2 clearly demonstrates a mediastinal mass. The clinical presentation of a mediastinal mass is often non-specic, although 60% of patients present with respiratory symptoms, typically cough, wheeze or dyspnoea, and are often misdiagnosed as having an upper respiratory tract infection or asthma. Approximately 25% of

PAEDIATRICS AND CHILD HEALTH 17:1

35

r 2007 Elsevier Ltd. All rights reserved.

SELF-ASSESSMENT

patients are asymptomatic, the mass being found incidentally. Because of the relatively small space in the mediastinum, children are much more likely than adults to present with acute upper airway obstruction. Important differentiating features are the presence of physical signs such as decreased air entry, stridor, tracheal deviation, superior vena cava syndrome or palpable lymphadenopathy. Supraclavicular lymph nodes are particularly worrying as they are strongly suggestive of malignancy and are an indicator for an early lymph node biopsy.7 Supraclavicular lymph nodes have been found to be a signicant predictor of malignancy, especially when associated with an abnormal chest X-ray.8 The most important initial investigation is a chest X-ray, and in most series all of the mediastinal masses can be detected on chest X-ray. The risk of malignancy in mediastinal tumours is high in children up to 75% and the most common diagnoses are lymphoma followed by a neurogenic mass, such as a neuroblastoma. Younger children are more likely to present with a neuroblastoma than a lymphoma, which is a more common nding in adolescents. Urinary catecholamines are generally raised in neuroblastoma, and lactate dehydrogenase (LDH) can be high in lymphoma. Mediastinal germ cell tumours are the next most common group of tumours, generally yolk sac tumours in children, followed by sarcomas. Alphafetoprotein and BHCG measurements are useful in identifying a germ cell tumour. The most frequently occurring benign masses are enteric duplications, ganglioneuromas, bronchogenic cysts, haemangiolymphangiomas, thymic cysts and teratomas. Infections, such as tuberculosis, actinomycosis, lung abscesses, fungal infections and rarely cytomegalovirus, can also present as mediastinal masses. The learning point from this case is to be aware of other diagnoses in children that cause respiratory symptoms and to look for other clues to the diagnosis, particularly before prescribing steroids, as well as to be prepared to investigate further should symptoms persist or not resolve as expected. Most lymphomas are generally very steroid-responsive, and children can show rapid resolution of their symptoms with steroid treatment. Airway compression is often a feature of mediastinal masses and can be life-threatening, particularly on induction of anaesthesia; therefore the most appropriate course of action is pretreatment with steroids to reduce the size of the mass prior to an anaesthetic. Nebulised adrenaline is used only as an emergency rescue medication in this setting and is unlikely to have much of an effect as the obstruction is physical. Answer 5 1. c Fat embolism syndrome. 2. Typical clinical features include a rapid onset within 72 hours of an insult, restlessness with vague chest pain, persistent tachycardia with respiratory distress, a high

spiking fever and features of neurological dysfunction including agitation, delirium, seizures or coma. Up to 50% of patients develop short-lived, reddish-brown, non-palpable petechiae over the upper body, particularly in the axillae, within 2436 hours of the insult. 3. Treatment is supportive. Fat embolism syndrome was rst described as clinical entity in the 19th century. The most common causes include blunt trauma (90% of all cases being associated with long bone or pelvic fractures, as in this case with bilateral distal femoral fractures), orthopaedic procedures, parenteral lipid infusion and corticosteroid administration. The mechanical theory of fat embolism pathophysiology suggests that large fat droplets released into the venous system are deposited in the pulmonary capillary beds and travel through arteriovenous shunts to the brain. Lodged microvascular droplets produce local ischaemia and inammation, with a concomitant release of inammatory mediators and vasoactive amines, and platelet aggregation. The biochemical theory suggests that hormonal changes caused by a variety of insults induce a systemic release of free fatty acids as chylomicrons, which coalesce and create the pathophysiological changes described. The diagnosis is a clinical one, and there is no specic treatment; the outcome is dependent on the nature of the clinical insult associated with its development.

REFERENCES

1 2 3

Herwaldt BL. Leishmaniasis. Lancet 1999; 354: 11919. Bhattacharya SK, Sur D, Karbwang J. Childhood visceral leishmaniasis. Indian J Med Res 2006; 123: 3536. Minodier P, Retornaz K, Horelt A, Garnier JM. Liposomal amphotericin B in the treatment of visceral leishmaniasis in immunocompetent patients. Fundam Clin Pharmacol 2003; 17: 1838. Coppes MJ, Zandvoort SWH, Sparling CR, et al. Acquired von Willebrand disease in Wilms tumor patients. J Clin Oncol 1992; 10: 4227. Michiels JJ, Budde U, van der Planken M, van Vliet HH, Schroyens W, Berneman Z. Acquired von Willebrand syndromes: clinical features, aetiology, pathophysiology, classication and management. Best Pract Res Clin Haematol 2001; 14: 40136. Maas MH, Cransberg K, van Grotel M, Pieters R, van den HeuvelEibrink MM. Renin-induced hypertension in Wilms tumor patients. Pediatr Blood Cancer 2006, June 22 [Epub ahead of print]. Karadeniz C, Oguz A, Ezer U, Ozturk G, Dursun A. The etiology of peripheral lymphadenopathy in children. Pediatr Hematol Oncol 1999; 16: 52531. Soldes OS, Younger JG, Hirschl RB. Predictors of malignancy in childhood peripheral lymphadenopathy. J Pediatr Surg 1999; 34: 144752.

PAEDIATRICS AND CHILD HEALTH 17:1

36

r 2007 Elsevier Ltd. All rights reserved.

SELF-ASSESSMENT

Self-assessment
Questions
Question 1 A 12-year-old boy is referred to your general paediatric clinic after 5 weeks of hot, swollen, painful joints rst shoulders and knees, and now right wrist and ankle. He has been tired and generally unwell, and unable to manage full weeks at school. He has had a orid rash, described as red, blotchy and nonitchy; it is usually on the trunk, but can appear anywhere. He has been walking with a limp for a few weeks because of the pain, and says that it is particularly difcult to walk normally in the mornings. His GP examined him 2 weeks into the illness and found a palpable spleen. The boy had severe tonsillitis 2 months ago and could not attend school. He did not receive antibiotics. There is no past or family history to note. On examination, you nd a generally well-looking boy who is well grown, in the 75th centile. Pulses are normal, but there is a soft systolic murmur. There is no hepatosplenomegaly, lymphadenopathy or nodules. The right wrist, right knee and right ankle are hot and swollen with restricted movement and there is a patellar tap. He has a blanching erythematous rash on his abdomen (Figure 1). (a) What is the most likely diagnosis? Systemic arthritis (systemic juvenile idiopathic arthritis, JIA) Rheumatic fever Post-streptococcal reactive arthritis Leukaemia Oligoarthritis What investigations would you perform to make your diagnosis? Echocardiography Radiography and ultrasonography or MRI of affected joints Autoantibody screen, antinuclear antibody (ANA), rheumatoid factor Throat swab, antistreptolysin-O test (ASOT) and antiDNaseB Full blood count (FBC), ESR, C-reactive protein (CRP), blood lm How would you manage him? High-dose non-steroidal anti-inammatory drug (NSAID) Penicillin then prophylaxis and high-dose NSAID High-dose NSAID and corticosteroid joint injections under general anaesthesia Intravenous methylprednisolone and NSAID Bone marrow aspirate On review 2 weeks later, his limping has worsened and he is complaining of pain in his right calf. The calf is hot and (d) swollen with some mild pitting ankle oedema, and there is swelling at the back of the knee. What is your differential diagnosis and how will you investigate this further?

Question 2
A 6-month-old baby is admitted with a 1 day history of worsening rash. He was treated for a respiratory tract infection 3 weeks ago with 3 days of prednisolone, beclomethasone and salbutamol inhalers and 1 week of amoxicillin. He had 2 days of fever 4 days ago, but this has now resolved. He seemed well, but developed facial and ear swelling 1 day ago; the rash appeared on his right cheek, spreading rapidly over his face, ears and neck. The lesions are discrete, circular, erythematous and non-blanching. They appear to be increasing in size, with some central necrosis. The parents say that his hands and feet have started to swell and the rash is appearing there also (Figures 2 and 3). On examination, he is interactive and well perfused, with a pulse of 110. His temperature is 36.8 1C. There is no lymphadenopathy, hepatosplenomegaly or conjunctivitis. (a) What is the diagnosis? Meningococcal sepsis Kawasaki disease Stage 4 s neuroblastoma (blueberry mufn) Erythema multiforme Acute haemorrhagic oedema of infancy What investigations are necessary for the diagnosis? (Choose as many as appropriate.) None FBC ESR CRP Blood culture PCR for meningococcus Throat swab Urinary vanillylmandelic acid (VMA):creatinine ratio Echocardiography Antineutrophil cytoplasm antibodies Abdominal ultrasonography What is the management? Intravenous ceftriaxone Observation Refer to dermatologist for skin biopsy Aspirin, intravenous immunoglobulin and echocardiography Paediatric oncology referral

(b)

(b)

(c)

(c)

Question 3
Kate Armon Paediatric Consultant at the Norfolk and Norwich University Hospitals NHS Trust, Norwich, Norfolk, UK.

A 15-month-old girl is admitted as an emergency with a 2 week history of worsening respiratory symptoms. She started with

PAEDIATRICS AND CHILD HEALTH 17:3

111

r 2007 Elsevier Ltd. All rights reserved.

SELF-ASSESSMENT

rhinorrhoea and cough for approximately 1 week and was treated with amoxicillin. She appeared to improve before deteriorating with increasing lethargy, poor feeding and intermittent grunting. There have been no recent fevers or rashes. She has had loose stools for the last 24 hours, and has seemed to be in distress and pain, being unable to sleep unless semi-reclining. Her parents recall a febrile illness with bright red cheeks 1 month ago, and she has not seemed well since then. Examination reveals a lethargic, pale and distressed toddler with respiratory rate 46, grunting respiration, minimal recession, good air entry, no added sounds, heart rate 152 and capillary rell 12 seconds. There is a large, rm abdominal mass in the right upper quadrant that you cannot get above, measuring 5 cm below the costal margin. (a) What is the most likely diagnosis? Neuroblastoma with hepatic metastasis Cystic brosis Pneumonia and hyperination Viral myocarditis Dilated cardiomyopathy Which of the following investigations are necessary to conrm your diagnosis? (Choose as many as appropriate.) Echocardiography Viral PCR for parvovirus, coxsackievirus and enteroviruses, and viral serology Abdominal ultrasonography Urinary VMA Chest radiograph Blood cultures ECG Blood gases Sweat test Trial of bronchodilator ECG and echocardiography of other family members What is the management? Broad-spectrum intravenous antibiotics Intubation and ventilation Analgesia, abdominal ultrasonography Nil by mouth, maintenance intravenous uids Facial oxygen and furosemide, 12 mg/kg i.v.

Figure 1

(b)

(c)

Figure 2

Question 4
A 12-year-old girl is sent to your assessment unit with a 5 week history of general malaise, worsening headache, joint pains, nausea with weight loss of about 1 kg, shortness of breath on minimal exertion and fevers, associated with sweating and a 3 day history of intermittent vomiting. There is no past or family history of note. Direct questioning reveals she always wears gloves in winter as her ngers turn white in the cold, and she has intermittent mouth ulcers. On examination, her temperature is 39 1C and there is mild cervical lymphadenopathy. Her cardiovascular, respiratory and neurological systems are unremarkable. There is mild livedo reticularis on her arms and generalized dry skin, with noticeable palmar erythema. Musculoskeletal examination

Figure 3

PAEDIATRICS AND CHILD HEALTH 17:3

112

r 2007 Elsevier Ltd. All rights reserved.

SELF-ASSESSMENT

shows reduced hand grip bilaterally with poor exion of the proximal interphalangeal (PIP) joints but no denite effusions. There is no mouth ulceration. The spleen tip is palpable. The results of initial investigations are as follows: Haemoglobin WBC Platelets ESR CRP Liver function tests Urea and electrolytes Creatine kinase PaulBunnell test Blood cultures Viral serology Lumbar puncture (a) 9.8 g/dL (normochromic, normocytic) 2.1 109/L, neutrophils 1.0 109/L, lymphocytes 0.8 109/L 105 109/L 97 mm/hour o3 Normal, except alanine aminotransferase 103 U/L (normal 050) Normal 65 U/L (normal 55170) Negative Negative Negative Normal

Answers
Answer 1 (a) Post-streptococcal reactive arthritis. Systemic JIA less likely as no fever, atypical rash. Rheumatic fever does not full criteria. Leukaemia should be considered; arthritis is seen on examination. Oligoarthritis malaise and rash not associated with oligoarthritis. Throat swab, ASOT and anti-DNaseB. Echocardiography important but will not conrm diagnosis. Radiography and ultrasonography or MRI of affected joints may conrm effusions/synovitis but does not give diagnosis. Autoantibody screen, ANA, rheumatoid factor unhelpful in diagnosis; evidence of streptococcal disease. FBC, ESR, CRP, blood lm all useful but nonconrmatory. Penicillin then prophylaxis and high-dose NSAID. High-dose NSAID not complete answer. High-dose NSAID and corticosteroid joint injections under general anaesthesia he may require corticosteroid injections, but the joints may settle on NSAID alone. Intravenous methylprednisolone and NSAID not necessary. Bone marrow aspirate no help, simply showing reactive picture. Bakers cyst most likely, deep vein thrombosis less likely; ultrasonography.

(b)

(c)

(b)

(c)

(d)

What is your differential diagnosis? Systemic lupus erythematosus (SLE) Dermatomyositis Leukaemia Systemic-onset juvenile idiopathic arthritis Cerebral abscess Which investigation will conrm your diagnosis? MRI of the brain with contrast MRI of the quadriceps muscles Blood lm, bone marrow aspirate ANA, double-stranded DNA (dsDNA) ANA, rheumatoid factor Choose one or more further important investigations. Bone marrow trephine and aspirate Autoantibody screen Lung function testing Muscle biopsy. Echocardiography Urinalysis Blood pressure MRI of the brain with contrast Lupus anticoagulant Coombs test Video uoroscopy Complement (C3, C4 and CH50) Immunoglobulins Bone marrow aspiration reveals a reactive marrow with evidence of haemophagocytosis. ANA enzyme-linked immunosorbent assay is 4.1 (positive 41.0) and dsDNA is 98 IU/mL (positive 415). The extractable nuclear antigen prole is positive for anti-RNP and anti-Sm, but otherwise negative. What is the diagnosis? Explain the bone marrow ndings.

(d)

The diagnosis is post-streptococcal arthritis. He has a 5 week history of signicant inammatory joint pain with joint heat, swelling, effusion and restricted movement (conrming arthritis rather than arthralgia) on examination. He has been generally unwell and has a rash, but there is no clear history of fever. The rash has annular-looking lesions, and though this photograph was taken towards the end of his illness when the rash was less orid, it ts with erythema marginatum. Blood tests showed haemoglobin 11.9 g/dL, WBC 8.5 109/L, platelets 386 109/L, ESR 94 mm/ hour, ASOT 400 IU/mL, anti-DNase B 41440 IU/mL, CRP 20 mg/ L, rheumatoid factor 17 IU/mL (normal 020) and ANA 0.7 IU/mL normal (negative). Echocardiography was normal. Systemic arthritis has a prevalence of approximately 0.5/ 100,000 and an average age of onset of 5 years,1 but can occur in any age group. Signicant manifestations of the disease are early fever and rash, with marked malaise. The fever usually occurs once (or twice) per day (quotidian pattern) and persists for at least 2 weeks.2 The rash is itting, typically worse with the fevers and is described as salmon pink macules exhibiting the Koebner phenomenon. The arthritis often appears later in the disease. Splenomegaly is seen in about 30% of cases and is usually mild. Pericarditis and pericardial effusions can occur. The absence of documented fever and the typical rash makes the diagnosis unlikely in this case. In the revised Jones criteria for the diagnosis of acute rheumatic fever (ARF), the diagnosis is likely with two major

PAEDIATRICS AND CHILD HEALTH 17:3

113

r 2007 Elsevier Ltd. All rights reserved.

SELF-ASSESSMENT

criteria or one major criterion and two minor criteria and documentation of preceding group A streptococcal infection. The major criteria are carditis (new or changing murmurs, cardiomegaly/congestive heart failure, pericarditis), migratory polyarthritis, chorea, erythema marginatum and subcutaneous nodules. The minor criteria are fever, arthralgia, elevated ESR and CRP, prolonged PR interval on ECG and a history of a prior attack of rheumatic fever. The arthritis in ARF is typically migratory; different joints are affected from day to day. The joints are often extremely painful and the arthritis rarely continues for more than 3 weeks untreated.3 It generally occurs 13 weeks after the streptococcal infection. The diagnosis of post-streptococcal reactive arthritis is made in children with arthritis following a documented group A Streptococcus infection who do not full the criteria for ARF. Their arthritis is non-migratory and can persist for 5 days to 6 months. There is debate about whether it is a separate entity from ARF, since the risk of a subsequent episode of ARF with carditis is relatively high (67%) in this population.4 It is important to perform echocardiography in these children. Treatment with 10 days of penicillin is recommended. Debate continues about the length of prophylaxis (12 years or indenitely) against further attacks. Many infections can cause reactive arthritis, the commonest being enteric pathogens (Salmonella, Shigella, Yersinia, Campylobacter), sexually transmitted organisms (Chlamydia trachomatis), parvovirus B19 and Borrelia burgdorferi (Lyme disease). Malignancy should be considered in children with fever, malaise, hepatosplenomegaly and joint pain. If the diagnosis is in doubt, an FBC and lm should be performed, progressing to bone marrow aspirate if necessary. Synovial cysts occur most commonly and dramatically in systemic arthritis, but can occur in any type of arthritis. Cysts in the popliteal fossa (Bakers cyst) are a posterior out-pouching of the synovial cavity, enlarging by a ball-valve effect, and can extend into the calf muscle and cause disruption of normal venous circulation, leading to painful leg swelling and oedema. They can rupture, with a sudden sharp pain in the calf muscle and swelling, followed by echymoses around the maleolae.5 Ultrasonography showed a 10 4 cm Bakers cyst with synovial hypertrophy and no evidence of thrombus in the femoral or popliteal veins.

Bamzai A, Kreiger M, Kretschmer RR. Synovial cysts in juvenile rheumatoid arthritis. Ann Rheum Dis 1978; 37: 1013.

Answer 2
(a) Acute haemorrhagic oedema of infancy the history and rash are typical of this condition. (b) None. (c) Observation. Acute haemorrhagic oedema of infancy (also known as Finkelsteins or Seidlmayers disease) is a benign self-limiting disease in children under 2 years of age. There may be a prodromal illness followed by non-pitting oedema in the extremities and symmetrical large purpuric lesions said to have a cockade pattern.1 There may be central necrosis. It mainly affects the face and extremities, sparing the trunk. Systemic features such as bloody stools or renal involvement occur very rarely, but have led to suggestions that this condition represents HenochScho nlein purpura (HSP) in infants.2 This patient developed vomiting and diarrhoea (one stool was bloody) 3 days later, which was thought most likely to be infective, though no viral or bacterial cause was found from stool specimens. If a skin biopsy is taken, leucocytoclastic vasculitis is seen (as in HSP). The condition usually resolves within 3 weeks. The diagnosis is made by recognition of the typical features, and no investigations are strictly necessary. However, the condition is considered to be a diagnosis of exclusion and a period of observation may be required, or some differentiating blood tests to reassure both clinician and parents.

REFERENCES
1 Liu AJW, Hogan P, Nanan R. Acute heamorrhagic oedema of infancy. Images in paediatrics. Arch Dis Child 2006; 91: 382. Casken H, Odabas D, Kosem M et al. Report of eight infants with acute infantile hemorrhagic edema and review of the literature. J Dermatol 2002; 29: 2905.

REFERENCES
1 Malleson PN, Fung MY, Rosenberg AM. The incidence of pediatric rheumatic diseases: results from the Canadian Pediatric Rheumatolgoy Association Disease Registry. J Rheumatol 1996; 23: 19817. Petty RE, Southwood TR, Manners P, Baum J, Glass DN, Goldenberg J, He X. International League of Associations for Rheumatology classication of juvenile idiopathic arthritis: second revision, Edmonton, 2001. J Rheumatol 2004; 31: 3902. Szer IS. Acute inammatory rheumatic syndromes. In: Szer IS, Kimura Y, Malleson PN, Southwood TR. Arthritis in children and adolescents. Oxford: Oxford University Press; 2006. Shulman ST, Ayoub EM. Poststreptococcal reactive arthritis. Curr Opin Rheumatol 2002; 14: 5625.

Answer 3
(a) Viral myocarditis the child is in cardiac failure; from the history, it is most likely to be caused by infection. (b) Echocardiography; viral PCR for parvovirus, coxsackievirus and enteroviruses, and viral serology; chest radiograph; ECG other investigations may be helpful but are optional. (c) Facial oxygen and furosemide, 12 mg/kg i.v. Four doctors had seen this patient on four separate occasions before this admission and diagnosis. Their diagnoses had been respiratory upper respiratory tract infection, pneumonia and asthma. At this attendance, the patient was initially thought to be in pain and distress (causing the grunting respirations) due

PAEDIATRICS AND CHILD HEALTH 17:3

114

r 2007 Elsevier Ltd. All rights reserved.

SELF-ASSESSMENT

to rapid liver capsule enlargement, before cardiac failure was diagnosed. On repeat clinical examination, she was found to have a gallop rhythm. It is unusual to see cardiac failure presenting at this age and all too easy to diagnose respiratory illness. The differential diagnosis in this case is acute myocarditis (commonly virus mediated), idiopathic dilated cardiomyopathy or familial cardiomyopathy. It is extremely important to look for causative organisms by viral PCR and serology at presentation. Coxsackieviruses and echoviruses are the commonest. In this case, parvovirus was thought to be the most likely cause, though unconrmed. Viral myocarditis has a better prognosis than dilated cardiomyopathy. A study of 62 Finnish children with idiopathic dilated cardiomyopathy at a mean follow-up of 4 years makes sombre reading: 16% recovered, 27% had residual disease, 6.4% underwent heart transplantation and 50% died.1 If no causative organism is found, it is important to screen the family for familial disease using ECG and echocardiography. Treatment comprises acute supportive care (intensive care as necessary), diuretics (both loop and thiazide) and the introduction of angiotensin-converting enzyme (ACE) inhibitors, slowly, in the hospital environment because of rst-dose hypotension. ACE inhibitors are now considered to be the cornerstone of care and there is a paediatric suspension of captopril.2 Aspirin is added if left ventricular function is severely impaired. There is some evidence of benet from carvedilol (a third-generation bblocker), but this is not proven.3 Immunosuppressive therapy in the form of prednisolone (7 intravenous immunoglobulin) is benecial for immune-mediated myocarditis (e.g. rheumatic fever, SLE).

REFERENCES
1 Arola A, Tuominen J, Ruuskanen O, Jokinen E. Idiopathic dilated cardiomyopathy in children: prognostic indicators and outcome. Pediatrics 1998; 101: 36976. Westaby S, Franklin O, Burch M. new developments in the treatment of cardiac failure. Arch Dis Child 1999; 81: 2767. Mir TS, Laer S, Eiselt M et al. Effect of carvedilol on QT duration in paediatric patients with congestive heart failure. Clin Drug Invest 2004; 24: 915.

Answer 4
(a) Systemic lupus erythematosus. Dermatomyositis no muscle weakness, poor grip is secondary to arthritis in the ngers, no heliotrope rash. Leukaemia blood results are more typical of an autoimmune condition. Systemic-onset JIA age, sex and clinical ndings are more typical of SLE. Cerebral abscess would not account for the joint symptoms/signs. ANA, dsDNA. Muscle biopsy, MRI of the brain with contrast, and video uoroscopy are not required.

(b) (c)

This child presented with a multisystem disorder and thorough investigation is mandatory. Bone marrow trephine and aspirate important to exclude leukaemia. Autoantibody screen aids diagnosis. Lung function testing pulmonary disease is common in children with SLE; in a study of 22 children, 17 had respiratory symptoms, of which one-half had abnormalities on chest radiography;1 pleural effusions, pleuritis, acute and chronic pneumonitis and pulmonary haemorrhage may occur. Muscle biopsy not necessary. Echocardiography pericarditis occurs in up to 30% of children with acute SLE, and may be clinically silent; myocarditis and valvulitis also occur. Urinalysis lupus nephritis is probably present to some degree in all children with SLE and is a major determinant of the long-term outcome of this disease; microscopic haematuria followed by proteinuria are the most common initial manifestations of nephritis.2 Blood pressure measurement necessary in any multisystem disorder. MRI of the brain with contrast not required; CNS involvement in SLE is common, the most frequent manifestation being headache; further investigation of the CNS (EEG, lumbar puncture, MRI) may be necessary with the development of additional symptoms (seizures, cognitive impairment, psychoses, chorea). Lupus anticoagulant coagulation abnormalities associated with lupus may not be detected on a standard clotting screen and more detailed testing is required; antiphospholipid antibodies (e.g. anticardiolipins) should be measured as their presence may indicate an increased tendency to thrombotic events such as venous or arterial thrombosis. Coombs test the anaemia seen in SLE may be due to anaemia of chronic disease, Coombs-positive haemolytic anaemia (if severe, with jaundice) or associated with cold agglutinins. Video uoroscopy unnecessary. Complement (C3, C4 or the total haemolytic complement, CH50) CH50 is abnormally low in approximately 90% of children with lupus nephritis; low C4 is a useful indicator of active nephritis in SLE if the patients baseline level is known. Immunoglobulins immunodeciencies associated with increased risk of developing SLE include low IgA, IgG and IgG subclasses, and any of the complement components. The diagnosis is systemic lupus erythematosis, a multisystem autoimmune disease characterized by widespread inammation of blood vessels and connective tissues and by the presence of ANAs, especially to dsDNA. The prevalence in children is estimated to be 0.36/100,000. In adults in the UK, the prevalence in Asians is three times greater than that in Caucasians, and that in Afro-Caribbeans is almost six times greater. It often presents in young women and mimics many other conditions. The predominant symptom is lethargy with aches and pains. These symptoms are common and lead to delay in diagnosis in many cases. Teenagers are often labelled with chronic fatigue syndrome/myalgic encephalomyelitis or glandular fever before the correct diagnosis is made. Fever, weight loss and lymphadenopathy are additional common, non-specic symptoms.

PAEDIATRICS AND CHILD HEALTH 17:3

115

r 2007 Elsevier Ltd. All rights reserved.

SELF-ASSESSMENT

In 1982, the American College of Rheumatology (ACR) established criteria for the diagnosis of SLE. These were primarily established as a classication tool for research and have been criticized for not identifying patients early enough in the course of the disease. They have been validated in children3 but the 1997 update awaits paediatric validation. The child must have had at least four of these 11 features at any time since the onset of the disease: Mucocutaneous lesions Buttery or Malar rash xed red rash over the cheeks 2. Discoid lupus red patches of skin associated with scaling and plugging of the hair follicles 3. Photosensitivity rash after exposure to sunlight 4. Oral or nasopharyngeal ulcers usually painless Musculoskeletal manifestations 5. Arthritis (non-erosive) Serositis 6. Pleuritis (pleuritic pain/rub or pleural effusion), pericarditis or peritonitis Renal disease 7. Proteinuria (40.5 g/day) or cellular casts Haematological abnormality 8. Haemolytic anaemia (with reticulocytosis), leucopenia (o4 on two or more occasions), Lymphopenia (o1.5 on two or more occasions) or thrombocytopenia (o100) Immunological abnormalities 9. Positive uorescence test for ANA 10. LE cells, anti-dsDNA antibodies, anti-Sm antibodies or chronic false-positive test for syphilis Neurological manifestations 11. Psychosis or convulsions both in absence of known metabolic derangement or precipitating drugs. Cerebral arteritis, transverse myelitis and peripheral neuropathy are also suggestive of SLE but are not part of the ofcial ACR criteria. This patient presented fairly typically. The initial investigations show pancytopenia with marked lymphopenia. The ESR is raised but the CRP is normal, typical of SLE and unusual in systemic JIA, dermatomyositis and polyarticular JIA, when the CRP is often also raised. The arthritis in SLE is common and manifests as pain, morning stiffness and joint swelling, all of which may suggest polyarticular JIA, but the pain is often disproportionate to the degree of arthritis. The swelling is often transient and not present at the time of examination. An inability to claw the ngers fully (ex the distal interphalangeal and PIP joints with extended metacarpophalangeal joints) such that the nger tips touch the palms is common. Rheumatoid factor can be demonstrated in the serum of about 20% of children with SLE, but does not correlate with joint destruction. The antibody to native (double-stranded) DNA is 70.8% sensitive for active SLE and 93.2% specic. Anti-Sm (Smith) antibodies are also highly specic for SLE. Anti-RNP antibodies suggest an overlap condition and can be present 1.

when Raynauds phenomenon is a feature (though this case reported white ngers, she did not exhibit the three phases of Raynauds that are required to make the diagnosis: pallor, then cyanosis, then hyperaemia). Muscle pains are common in childhood SLE, and occasionally there is associated muscle inammation with weakness and raised muscle enzymes, though this is not typical. Weight loss is a feature of SLE and systemic JIA, but is not usually severe. Children presenting with musculoskeletal complaints and weight loss should also be evaluated for inammatory bowel disease. This child did not have typical mucocutaneous manifestations of lupus, though there was a history of mouth ulceration and she had palmar erythema, which is less commonly seen in SLE. Haemophagocytosis in the bone marrow occurs rarely at presentation of SLE and should be treated aggressively. It appears to be on a spectrum of haemophagocytic disorders. A serious complication of childhood systemic inammatory disorders is macrophage activation syndrome, which is characterized clinically by haemorrhages and CNS dysfunction, and haematologically by low platelets, hypobrinogenaemia, raised liver transaminases, leucopenia and raised ferritin.4 Haemophagocytic histiocytes are seen in the bone marrow. It appears to be precipitated by infection or medication and must be treated aggressively with pulsed corticosteroid and cyclosporin A.5 Treatment of SLE is with corticosteroid, hydroxychloroquine, avoidance of sun exposure and sun block. The addition of other immunosuppressants as corticosteroid-sparing agents is dependent on the severity and the response of the disease to corticosteroid. This child was given methylprednisolone, 30 mg/kg i.v. daily for 3 days, followed by prednisolone, 2 mg/kg p.o., until the haematology ndings returned to clinically normal ranges, with C4 and dsDNA approaching normal and clinical resolution of symptoms. The prednisolone is then weaned slowly over several months. If weaning is too rapid, the disease may relapse. Symptoms of headache on corticosteroid withdrawal may also occur. Most clinicians use hydroxychloroquine as a baseline immunosuppressive agent in SLE.

REFERENCES
1 Delgado EA, Malleson PN, Pirie GE, Petty RE. The pulmonary manifestations of childhood onset systemic lupus erythematosus. Semin Arthritis Rheum 1990; 19: 285. Petty RE, Cassidy JT. Systemic lupus erythematosus. In: Petty RE, Cassidy JT. Textbook of pediatric rheumatology. 4th ed. Philadelphia: Saunders; 2001. Li SC, Imundo LF. Major rheumatic diseases. In: Szer IS, Kimura Y, Malleson PN, Southwood TR. Arthritis in children and adolescents. Oxford: Oxford University Press; 2006. Ravelli A, Magni-Manzoni S, Pistorio A et al. Preliminary diagnostic guidelines for macrophage activation syndrome complicating systemic juvenile idiopathic arthritis. J Pediatr 2005; 146: 598604. Stephan JL, Kone-Paut I, Galambrun C et al. Reactive haemophagocytic syndrome in children with inammatory

PAEDIATRICS AND CHILD HEALTH 17:3

116

r 2007 Elsevier Ltd. All rights reserved.

SELF-ASSESSMENT

disorders. A retrospective study of 24 patients. Rheumatology 2001; 40: 128592.

Tan E, Cohen A, Fries J. The 1982 revised criteria for the classication of systemic lupus erythematosus. Arthritis Rheum 1982; 25: 12717.

FURTHER READING
Bosi Ferraz M, Goldenberg J, Hilario M et al. Evaluation of the 1982 ARA lupus criteria data set in pediatric patients. Clin Exp Rheumatol 1994; 12: 837.

PAEDIATRICS AND CHILD HEALTH 17:3

117

r 2007 Elsevier Ltd. All rights reserved.

SELF-ASSESSMENT

Self-assessment
Questions
Question 1 A 12-year-old girl presents with a short history of polyuria, polydipsia and weight loss. There is no past medical history of note, but there is a strong family history of diabetes on her fathers side (Figure 1). She is clinically well and not dehydrated. Her height is 148 cm (50th centile) and weight 35 kg (25th centile). Her blood pressure is within normal limits for her age. Initial investigations are as follows.
Random blood glucose Venous blood gas 24.2 mmol/l pH 7.34 Base excess 0.2 Bicarbonate 25.3 mmol/l pCO2 6.0 kPa Sodium 137 mmol/l Potassium 3.9 mmol/l Urea 3.5 mmol/l Creatinine 54 mmol/l 14.1% Glycosuria, no ketonuria Negative

Question 2 A 15-year-old girl is referred because of concern about short stature. Her growth chart is shown in Figure 2. She is otherwise well. She needed grommets as well as squint surgery as a younger child. She has not yet started her periods, though her mother reached menarche by 13 years of age. The mean parental height centile is the 91st. Examination ndings are as follows:
Height Sitting height Weight Breast stage Pubic hair stage Cardiovascular examination Numerous pigmented naevi, hyperconvex nails 145 cm (0.4th centile) 79 cm (approximately 2nd centile) 53 kg (50th centile) 1 2 Normal

Electrolytes

HbA1C Urinalysis Islet cell antibodies

(a) What is the most likely diagnosis? Type 2 diabetes Diabetes insipidus Type 1 diabetes Maturity onset diabetes of the young (MODY) (b) What would be the most useful test to establish the diagnosis? Oral glucose-tolerance test Water-deprivation test Molecular genetic analysis Glutamic acid decarboxylase (GAD) antibodies (c) What would be the most appropriate treatment at presentation in this girl? Oral hypoglycaemic drug Diet modication Intravenous insulin and intravenous uids Subcutaneous insulin

(a) What is the most likely diagnosis? Constitutional delay in growth and puberty Turner syndrome Normal growth Growth hormone deciency Hypothyroidism (b) Which investigation would be most helpful in conrming the diagnosis? Bone age Thyroid function tests Chromosomal analysis Growth hormone-stimulation test (c) Which of the following is not a licensed indication for growth hormone? Chronic renal failure PraderWilli syndrome Achrondroplasia Small for gestational age Turner syndrome Question 3 A 3-week-old baby boy presents with 3 days of progressively worsening vomiting that at times is projectile. He is unsettled but not pyrexial and has no diarrhoea. He is the rst-born male in the family and was born at term, with a birth weight of 3.46 kg.He was previously well, and there is no family history of note. On examination, he appears unwell and wasted (weight 3.25 kg). There are no specic ndings on examination. He has normal male external genitalia. Observations and investigations are as follows.
Temperature Pulse Blood pressure Capillary rell Respiratory rate Saturation in air 36 1C 130 66/36 mmHg o2 s 34 99%

Drusilla Ferdinand MB CHB MRCPCH is Specialist Registrar at the Derbyshire Childrens Hospital, Derby City Hospital, Uttoxeter Road, Derby, UK. Nadya James MBBS BSC MRCPCH is Specialist Registrar at the Derbyshire Childrens Hospital, Derby City Hospital, Uttoxeter Road, Derby, UK. Julie Smith MA BM BCH MRCP(UK) MRCPCH is Consultant Paediatrician at the Derbyshire Childrens Hospital, Derby City Hospital, Uttoxeter Road, Derby, UK. E-mail: Julie.smith13@nhs.net

PAEDIATRICS AND CHILD HEALTH 17:4

147

r 2007 Elsevier Ltd. All rights reserved.

SELF-ASSESSMENT

Carcinoma 78 years

Died ?cause age 53 years

MI 45years, Diabetes 20 years

Started diet/tablets aged 20 years

Diagnosed age 19 years

Detected age 15 years after sister diagnosed

Patient

Figure 1 Family tree of patient 1 showing those affected with diabetes.

Haemoglobin White cell count Platelets Sodium Potassium Urea Creatinine Chloride Bicarbonate Calcium Glucose C-reactive protein

15.5 g/dl 10.8 109/l 443 109/l 117 mmol/l 7.3 mmol/l 11.8 mmol/l 52 mmol/l 90 mmol/l 18 mmol/l 2.9 mmol/l 4.5 mmol/l o2 mg/l

(a) What is the most likely diagnosis? Pyloric stenosis Acute gastroenteritis Salt wasting 21-hydroxylase deciency Acute renal failure Systemic sepsis (b) What additional investigations would conrm the diagnosis? Test feed 17OH progesterone Blood culture Abdominal ultrasonography Paired urinary and plasma osmolality (c) What emergency action is required now? Intravenous hydrocortisone bolus followed by 0.9% saline infusion Lumbar puncture followed by intravenous antibiotics Oral feeds and oral prednisolone Intravenous insulin-dextrose infusion Calcium resonium Question 4 A 15-year-old boy presents with an episode of involuntary jerking movements of his right arm lasting a few minutes. He

has been unwell for the preceding 3 weeks, with coryzal symptoms, sore throat and a headache. His previous medical history is unremarkable and illicit drug use is denied. Examination is normal, as are his renal prole, full blood count, C-reactive protein, glucose and calcium. He is observed overnight, with no further episodes, and is discharged the following day. He presents again 3 days later with worsening involuntary, non-rhythmical movements of his head, neck and right arm and leg. The movements become more obvious during anxiety, improve with distraction and disappear with sleep. The headache is unchanged. During this presentation, previous illicit drug use is disclosed, including cannabis, ecstasy and amphetamines. He denies any use over the last 7 weeks. Examination is otherwise normal. He is admitted for observation and continues to be symptomatic. Both CT of the head and EEG are normal.

(a) Which of the following is the most appropriate diagnosis? Partial seizures Parkinsons disease Sydenham chorea Tourettes syndrome Brain tumour He continues to be symptomatic. A urine toxicology screen is positive for cannabinoids. All blood tests are normal, except for a raised antistreptolysin-O test (ASOT). He is commenced on haloperidol, which improves his symptoms. Two days later, he complains of a strange sensation in his tongue. There are no abnormalities noted on examination. Two hours after this, he becomes distressed. His eyes are rolled up with involuntary protrusion of his tongue and a backward tilt to his head. He is able to make controlled movements with effort. All reexes are normal.

PAEDIATRICS AND CHILD HEALTH 17:4

148

r 2007 Elsevier Ltd. All rights reserved.

SELF-ASSESSMENT

Age (years) 9 10 11 12 13 14 15 16 17 18 19 Height (centimetres) 205 200 195 190 185 180 175 170 165 160 155 150 Weight (kilogrammes) 110 105 100 95 90 85 80 75 70 65 60 55 50 45 40 35 30 25 20
Figure 2

(b) Which of the following is now the most appropriate diagnosis? Partial seizures Complex seizures Dystonic reaction Supratentorial factors Tardive dyskinesia (c) Which of the following drugs would be the most appropriate to treat his ongoing choreoform movements? Penicillin Carbamazepine Lamotrigine Ethosuximde Erythromycin

Question 5 A 5-month-old boy is referred to the clinic by his general practitioner with a sacral dimple. His antenatal history is unremarkable. His development and growth are normal. On examination, he is noticed to be tachypnoeic with bilateral equal air entry. His apex beat is displaced to the right, with normal heart sounds on auscultation and good peripheral pulses. The remaining examination is normal apart from a blind-ending sacral dimple. His chest radiograph is shown in Figure 3. (a) What are the most striking abnormalities on the radiograph? Consolidation of the right upper and middle lobes and mediastinal shift Dextrocardia Right-sided pleural effusion and mediastinal shift

PAEDIATRICS AND CHILD HEALTH 17:4

149

r 2007 Elsevier Ltd. All rights reserved.

SELF-ASSESSMENT

Figure 3

Agenesis of right upper and middle lobes with mediastinal shift Left-sided tension pneumothorax with mediastinal shift (b) What are the arrows indicating? Air bronchogram Right heart border Lung edge Diaphragmatic hernia Scimitar vein (c) Which further investigation would best aid clarication of the diagnosis? CT of the chest Magnetic resonance angiography ECG Abdominal ultrasonography Echocardiography

Answers
Answer 1

cell antibodies 6879%), though absence of antibodies does not exclude type 1 diabetes. Measurement of GAD antibodies may be useful when there is diagnostic doubt. Ketonuria is often present at diagnosis and there is often a family history of diabetes or other autoimmune conditions. Type 2 diabetes is characterised by insulin resistance and is associated with obesity. There may be other features of the metabolic syndrome (acanthosis nigricans, hypertension, hyperlipidaemia) and a family history of diabetes. The child may be asymptomatic or have features of polyuria and polydipsia. Ketonuria may be present. MODY is a heterogeneous group of disorders caused by defects in single genes resulting in beta cell dysfunction. It is characterised by beta cell dysfunction, young onset (o25 years) and autosomal dominant inheritance. Obesity is not a feature. The diagnosis should be considered in non-obese patients with a two- or three-generation history of diabetes on one side of the family. Molecular genetic analysis identies the gene defect. Prognosis and management depend on the defect. Some patients do not require insulin. The term latent autoimmune diabetes refers to a group of patients who have antibodies consistent with type 1 diabetes but whose condition initially behaves as non-insulin requiring. Diabetes is associated with a number of other disorders (e.g. cystic brosis). Diabetes insipidus is also characterised by polyuria, polydipsia and weight loss, but is due to a deciency of (central diabetes insipidus) or renal insensitivity to (nephrogenic diabetes insipidus) vasopressin (antidiuretic hormone). It is not associated with hyperglycaemia, glycosuria or raised HbA1C. Clinically, polydipsia becomes a prominent feature, with the patient waking at night to drink and drinking any available uid. If diabetes insipidus is suspected, investigations should include paired serum and urine osmolality and consideration of a water deprivation test to differentiate between the two causes. Management at diagnosis unwell or ketotic children should be started on insulin at diagnosis. The diagnosis and management can then be reviewed if there are atypical features of diabetes (low insulin requirement, no subsequent ketonuria, strong family history of non-type 1).

FURTHER READING
(a) Maturity inset diabetes of the young (MODY) (b) Molecular genetic analysis (c) Subcutaneous insulin The WHO denes diabetes as venous plasma glucose 7 mmol/l or more fasted or 11.1 mmol/l or more unfasted on more than one occasion. Most cases (99%) are type 1 diabetes, but a number of other types of diabetes are now recognised in children. These subtypes are rare, and correct diagnosis is important as their management and clinical course differ. Clinical features and laboratory investigations help to differentiate the various types. Type 1 diabetes is an autoimmune disorder characterised by beta cell destruction and dependence on insulin. Pancreatic antibodies are often present at diagnosis (GAD-65 2576%, islet

Diagnosis and management of monogenic diabetes in children. /www.diabetes.genes.co.ukS Porter JR, Barrett TG. Acquired non-type 1 diabetes in childhood: subtypes, diagnosis and management. Arch Dis Child 2004;89:113844.

Answer 2 (a) Turner syndrome (b) Chromosomal analysis (c) Achrondroplasia

PAEDIATRICS AND CHILD HEALTH 17:4

150

r 2007 Elsevier Ltd. All rights reserved.

SELF-ASSESSMENT

Turner syndrome is the triad of short stature, ovarian failure and dysmorphic features. It occurs due to loss of all or part of one of the X chromosomes. A proportion of patients are mosaic for the condition. The phenotype is variable. Short stature results from impairment of all phases of childhood growth. Mean birth length is 0.51.0 standard deviations below the mean. Growth rate declines during infancy and childhood and the pubertal growth spurt is absent. In the UK, the untreated nal height is approximately 142 cm. Leg length is affected more than sitting height; thus, the ratio of sitting to total height is slightly greater than average in this condition. Management is with growth hormone. For optimal results, this should be started by the time the child starts school (certainly no later than 8 years), or sooner if unduly short. In this case, there would be little benet from growth hormone at this age, and oestrogen therapy should be commenced to induce puberty. The role of oxandrolone (an anabolic steroid) in the management of short stature in Turner syndrome remains controversial. It is currently under investigation as part of the UK Turner study. Growth hormone is also licensed for growth hormone deciency, PraderWilli syndrome, chronic renal insufciency and children born small for gestational age. All but a small number of patients require oestrogen treatment for induction of puberty and long-term hormone replacement during adulthood. The optimal time for starting oestrogen replacement remains controversial, though it is usually started at around 13 years of age. The rationale for delaying it as long as possible is to prolong the period of growth before epiphsyeal fusion. Dysmorphic features include low hairline, widely spaced nipples, increased carrying angle of elbows, multiple pigmented naevi and hyperconvex nails. Other associated problems are congenital heart disease (bicuspid aortic valve, aortic stenosis, coarctation of the aorta), middle ear problems, squint, renal anomalies and specic learning difculties. Women with Turner syndrome require long-term follow-up. This is not only to supervise hormone replacement and advise on fertility issues, but for the early detection and treatment of various medical conditions. These include hypertension, aortic root dilatation (risk of death from aortic dissection/rupture), hearing loss and hypothyroidism. As both this patients parents are on the 91st centile, and the growth chart shows a tailing-off of growth rather than a consistent centile, normal growth with familial short stature is unlikely. Likewise, constitutional delay in growth and puberty does not t with her lack of pubertal signs when her mother was menstruating by the age of 13 years. Although hypothyroidism and growth hormone deciency remain a possibility, and can be evaluated by laboratory investigation, the constellation of symptoms and signs in this patient make Turner syndrome by far the most likely diagnosis.

Answer 3 (a) Salt wasting 21-hydroxylase deciency (b) 17OH progesterone (c) Intravenous hydrocortisone bolus followed by 0.9% saline infusion 21-hydroxylase (21-OH) deciency is the commonest adrenal enzyme deciency and accounts for 95% of cases of congenital adrenal hyperplasia. It results from mutations in the gene encoding P450c21. 21-hydroxlase is an enzyme involved in the metabolism of cholesterol. Without it, the adrenal cortex cannot synthesize cortisol or aldosterone; instead, synthesis is diverted to the androgenic steroids. 17OH-progesterone is elevated in plasma samples (usually 4100 nmol/l) and is the basis of diagnosis (Figure 4). A urinary steroid prole and genetic analysis further clarify the diagnosis. The clinical manifestations depend on the mutation of the P450c21 allele.

 Females with salt wasting 21-OH deciency present with


virilisation at birth, while males present from day 5 onwards with a salt-losing crisis featuring poor feeding, vomiting and weight loss. The diagnosis here is revealed by the hyponatraemic hyperkalaemic dehydration, without high bicarbonate. In pyloric stenosis, we might expect metabolic alkalosis with low potassium and high bicarbonate. Acute gastroenteritis rarely produces the marked metabolic derangement seen in this case history. We may expect raised white cell count and C-reactive protein and temperature instability if septic. Acute renal failure is also a possibility, but the creatinine is not sufciently elevated. Simple virilising 21-OH deciency is not salt wasting, and both boys and girls usually present later in childhood (usually 24 years) with evidence of excess androgens (enlarged clitoris or penis, pubic hair, tall stature and advanced bone age). Non-classic 21-OH deciency is seen in females, who present from adolescence onwards with features of androgenicity.

FURTHER READING
Donaldson MDC, Gault EJ, Tan KW, Dunger DB. Optimising management in Turner syndrome: from infancy to adult transfer. Arch Dis Child 2006;91:51320. Rongen-Westerlaken C, Rikken B, Vastrick P, et al. Body proportions in individuals with Turner syndrome. Eur J Pediatr 1993;152:81317.

Management of the salt-losing crisis involves replacing the missing cortisol with supraphysiological doses of glucocorticoids such as intravenous hydrocortisone to rescue the child from the adrenal crisis. In dehydrated children, a 0.9% bolus of saline followed by an infusion helps to correct the haemodynamic abnormalities and gradually restores serum sodium levels. If the child is hypoglycaemic, a bolus of 10% dextrose is required, followed by the addition of 10% dextrose to the 0.9% saline infusion. Intravenous hydrocortisone is then given every 6 hours initially. Oral uid and steroid replacement are not appropriate for children presenting in crisis. Although the potassium is high, it responds well to a uid bolus at commencement of hydrocortisone. Insulin-dextrose infusion may help reduce the potassium, but not the hypovolaemia or hypoglycaemia. Calcium resonium is not useful in the acute management of hyperkalaemia.

PAEDIATRICS AND CHILD HEALTH 17:4

151

r 2007 Elsevier Ltd. All rights reserved.

SELF-ASSESSMENT

Cholesterol

Pregnenolone

17OH-Pregnenolone

Dehydroepiandrosterone

Progesterone 21-Hydroxylase

17OH-Progesterone 21-Hydroxylase

Androstenedione

Testosterone Cortisol Aldosterone

Figure 4 Without the enzyme 21-hydroxylase, there is accumulation of progesterone and 17-hydroxyprogesterone, and decient aldosterone and cortisol synthesis.

Once the plasma sodium is normal and the baby is stable, oral hydrocortisone (in three divided doses), oral udrocortisone and oral sodium supplements can be commenced. Sodium supplements are usually stopped around the age of 12 months. Parents of children with 21-OH deciency should be educated in the management of intercurrent illness. If unwell, the usual dose of oral hydrocortisone should be doubled. If this is not tolerated (e.g. there is vomiting), the child should be given hydrocortisone intramuscularly and taken to the hospital for review. Intravenous uids and hydrocortisone may then be required.

FURTHER READING
Brook GD, Hindmarsh PC. Clinical pediatric endocrinology. Oxford: Blackwell Science; 2001. Migeon CJ, Wisniewski AB. Congenital adrenal hyperplasia owing to 21-hydroxylase deciency: growth, development, and therapeutic considerations. Endocrinol Metab Clin North Am 2001;30:193206. Speiser P ,White P. Congenital adrenal hyperplasia. N Engl J Med 2003;349:77688.

Answer 4 (a) Sydenham chorea (b) Dystonic reaction to haloperidol (c) Carbamazepine In the case described, the history of drug use and positive urinary cannabinoids are confounding factors. The presenting symptoms of chorea are not classically associated with drug withdrawal. Parkinsons disease can also be excluded; not only is this extremely rare in the paediatric population, but the main clinical features are tremor and rigidity. It is also progressive in nature. The presenting features closely resemble those of Tourettes syndrome, and indeed the symptomatic treatment is very similar. The distinguishing features in this case are that the movements are not voluntarily reproducible (as usual in Tourettes) and the raised ASOT, indicating a streptococcal infection.

Sydenham chorea (St Vitus dance) is the most common acquired chorea of childhood and a cardinal feature of rheumatic fever. Alone, it is enough to diagnose rheumatic fever. Chorea occurs in approximately 1020% of patients, with a female to male ratio of 2:1. Most patients present between the ages of 5 and 15 years, with a peak in adolescence. The underlying pathological process is thought to be an immune-modulated response to a group A b-haemolytic Streptococcus infection. Antineuronal antibodies cross-react with basal ganglial neurons, causing an imbalance in the dopaminergic, cholinergic and gamma-aminobutyric acid (GABA) systems. The features are an insidious onset of rapid, purposeless, non-repetitive, involuntary movements that are worse with stress and disappear with sleep. Hypotonia and muscular weakness can also be present. Symptoms can occur up to 6 months after the initial infection. The diagnosis is clinical, with most investigations unremarkable. There may be evidence of a previous streptococcal infection. The prognosis is good, with most cases resolving within 36 months. Severe cardiac involvement is rare in patients with chorea. Treatment of Sydenham chorea is symptomatic. Occasionally, sedatives such as diazepam are required if the movements are self-injurious. Dopaminergic blockers such as haloperidol are effective, but have a well-known side effect prole including tardive dyskinesia (which can persist following drug withdrawal) and dystonic reactions. Dystonic reactions are treated with antimuscarinic drugs such as procyclidine, which was used in this patient. Anticonvulsants such as carbamazepine and sodium valproate are thought to act on the GABA system and are also effective in controlling the choreoform movements. Penicillin is used to treat the source of the infection.

FURTHER READING
Behrman RE, Kliegman RM, Jenson HB (eds). Nelson textbook of pediatrics, vol. 590(2). 17th ed. Philadelphia: Saunders; 2004: p. 20201.

PAEDIATRICS AND CHILD HEALTH 17:4

152

r 2007 Elsevier Ltd. All rights reserved.

SELF-ASSESSMENT

Fenichel GM. Clinical paediatric neurology: a signs and symptoms approach. 3rd ed. Philadelphia: Saunders; 1997. p. 3067. McIntosh N, Helms PJ, Smyth R (eds). Forfar and Arneils textbook of pediatrics, 6th ed. Edinburgh: Churchill Libingstone; 2003. p. 961. Mohamed I, Moorjani B. Chorea in children. /www.emedicine.com/ neuro/topic644.htmS, accessed August 2006. National Institute of Neurological Disorders and Stroke. Sydenham chorea information page. /www.ninds.nih.gov/disorders/sydenham/ sydenham_pr.htmS, accessed August 2006. Sydenhams chorea. In: The Merck manual of diagnosis and therapy. Section 19, chapter 271. Neurological disorders. /www.merck.com/ mrkshared/mmanual/section19/chapter271/271a.jspS, accessed August 2006.

Answer 5 (a) Agenesis of right upper and middle lobes with mediastinal shift (b) Scimitar vein (c) Magnetic resonance angiography The chest radiography shows a marked shift of the mediastinal structures towards the right, which is not a nding of dextrocardia. There is opacication of the right upper and middle lobes not associated with air bronchograms, and a normally aerated lower lobe visible posteriorly. This makes the diagnoses of consolidation or pleural effusion unlikely. The left lung has no focal abnormality, though the pulmonary vasculature appears to be increased, thereby excluding pneumothorax. These appearances are consistent with agenesis of the right middle and upper lobes. The arrows are indicating the presence of a scimitar vein associated with these changes. Scimitar syndrome is a complex congenital lesion of the lung and associated vasculature. The classical ndings of scimitar

syndrome are a hypoplastic right lung with mediastinal shift and anomalous pulmonary venous drainage into the right atrium or inferior vena cava. The abnormal vein can produce a scimitar-shaped shadow on the chest radiograph. The presentation is variable, can occur at any age, including adulthood, and can be an incidental nding. In infancy, the course is usually more severe, with the commonest presentation being tachypnoea. This is an indication of the associated cardiac dysfunction and pulmonary hypertension. Investigations include echocardiography and cardiac catheterisation to delineate the vascular structure. Magnetic resonance angiography is the most accurate non-invasive investigation used to evaluate the vasculature and anatomy. Most asymptomatic patients have a normal lifespan with no complications. Patients that are symptomatic often have cardiac complications. The outcome of these cases depends on the feasibility of correcting the anomalies. Surgical techniques used include coil embolisation, pneumonectomy and complete repair. Lung transplantation is required rarely.

FURTHER READING
Gao YA, Burrows PE, Benson LN, et al. Scimitar syndrome in infancy. J Am Coll Cardiol 1993;22:87382. Huddleston CB, Exil V, Canter CE, Mendeloff EN. Scimitar syndrome presenting in infancy. Ann Thorac Surg 1999;67:1549. Khan MA, Torres AJ, Printz BF, Prakash A. Usefulness of magnetic resonance angiography for diagnosis of scimitar syndrome in early infancy. Am J Cardiol 2005;96:131316. rnberger V, Fenchel M, et al Scimitar syndrome: Kramer U, Do morphological diagnosis and assessment of hemodynamic signicance by magnetic resonance imaging. Eur Radiol 2003;S6:L147L150. Vanderheyden M, Goethals M, Van Hoe L. Partial anomalous pulmonary venous connection or scimitar syndrome. Heart 2003;89:761. Wallis C. Clinical outcomes of congenital lung abnormalities. Paediatr Respir Rev 2000;1:32835.

PAEDIATRICS AND CHILD HEALTH 17:4

153

r 2007 Elsevier Ltd. All rights reserved.

SELF-ASSESSMENT

Self-assessment questions

Questions
Case 1 A t, thriving 6-year-old boy is referred to outpatients by his GP. No family history of congenital heart disease exists. There is 2/6 ejection-systolic murmur at the upper sternal edge, with xedsplitting of the second heart sound. His ECG is shown below (Figure 1). 1. What are the ECG ndings? A Complete right bundle branch block B Partial right bundle branch block C Normal D Superior axis E Right axis deviation 2. Which one of the following is the likely diagnosis? A Primum ASD B VSD C Eisenmenger syndrome D Secundum ASD E Innocent murmur 3. Which one of the following would be the optimal management? A Discharge from clinic B Review in 6 weeks C Review in clinic in 1 year D Refer to cardiologist E Refer to cardio-thoracic surgeon Case 2 An 8-year-old girl presents with 1 week of pyrexia, vomiting and dizziness. During this time she has become increasingly breathless and has intermittent chest pain. She had a coryzal illness 4 weeks ago. Her ECG is shown below (Figure 2). 1. Which two of the following are seen in the ECG? A Widespread ST segment changes B Complete bundle branch block C Wenkebach phenomenon
Deepan Vyas is at the Departments of General Paediatrics, Birmingham Childrens Hospital, Steelhouse Lane, Birmingham B4 6NH, UK. Vin Diwakar is at the Departments of General Paediatrics, Birmingham Childrens Hospital, Steelhouse Lane, Birmingham B4 6NH, UK. Ashish Chikermane, MRCP MRCPCH is at the The Heart Unit, Birmingham Childrens Hospital, Steelhouse Lane, Birmingham B4 6NH, UK.

D Partial bundle branch block E First-degree heart block 2. Which one of these diagnoses correlates with the ECG? A Pericarditis B Myocarditis C Bacterial endocarditis D Pulmonary embolus E Myocardial infarction Case 3 A 13-year-old girl presents to the Emergency Department with intermittent palpitations. Over the last few months she has had three similar episodes with no neurological or cardiovascular compromise. Her ECG is shown below (Figure 3). 1. Which two of the following are seen in the ECG? A Prolonged PR interval B Decreased PR interval C Second-degree heart block D Delta wave E Digoxin reverse tick sign 2. Which one of the following is the likely diagnosis? A Digoxin overdose B Prolonged QT syndrome C WolfParkinsonWhite syndrome D Thyrotoxicosis E Sinus tachycardia Case 4 A 4-month-old baby was referred to the Paediatric Unit for failure to thrive. She had a dynamic precordium with a prominent ejection systolic murmur at the upper sternal edge radiating to the back. A chest X-ray showed an abnormal cardiac silhouette with reduced pulmonary vascularity. Her ECG is shown below (Figure 4). 1. Which three of the following can be seen in her ECG? A Right ventricular hypertrophy B Left ventricular hypertrophy C Abnormal T-wave D Peaked P waves E Bi-ventricular hypertrophy F Second-degree heart block G Superior axis deviation H Narrow QRS complex I Right bundle branch block J Prolonged QT interval K ST segment depression

PAEDIATRICS AND CHILD HEALTH 17:5

211

SELF-ASSESSMENT

V1

II

V2

III

V3

aVR

V4

aVL

V5

aVF

V6

Figure 1 A 6-year-old with an asymptomatic systolic murmur.

V1

V4

II V2 V5

III

aVR V3 V6

aVL

aVF

Figure 2 A 8-year-old girl with pyrexia, dyspnoea and chest pain.

I II

V4 R

V1 V4

III

V2 V5

aVR

aVL

V3 V6

aVF

Figure 3 A 13-year-old girl with intermittent palpitations.

PAEDIATRICS AND CHILD HEALTH 17:5

212

SELF-ASSESSMENT

V4 R I aVR V1 V4 V4 2

II

aVL

V2 V5

V5 2

III aVF

V3 V6 V6 2

Figure 4 A 4-month-old baby with failure to thrive and a systolic murmur (V4/2 V6/2 denotes halfvoltages).

2. Given her clinical ndings, which one of the following is the most likely diagnosis? A Total anomalous pulmonary venous drainage B Pulmonary stenosis C Tetralogy of Fallot D Ventricular septal defect E Ebstein anomaly

Most atrial communications spontaneously close in the rst year of life. In those that persist, most patients are asymptomatic until adulthood when they can present with dyspnoea, atrial arrhythmias (increasing risk with age up to 52% in the seventh decade) and pulmonary hypertension.

FURTHER READING Answers


Case 1 1. B Rate 100. Sinus rhythm. There is a partial right bundle branch block with the QRS axis at 901. 2. D ECG ndings are consistent with a secundum atrial septal defect (ASD). The presence of a pulmonary murmur with xed splitting of the second heart sound is characteristic of a large atrial shunt. A primum ASD would present with a superior axis, i.e., above 1801 (atrio-ventricular junction abnormality). 3. D The paediatric cardiologists used a percutaneous device to close the shunt. Atrial communications are usually of three types: 1. Primum defects are abnormalities in the formation of the atrio-ventricular junction. These range from a simple atrial shunt (primum ASD) to atrial and ventricular defects with associated abnormalities of the mitral and tricuspid valves (complete AVSD). 2. Secundum defects are due to a defect between the two atrial chambers, usually at the level of the oval foramen. 3. Patent oval foramen (PFO) is due to the failure of closure of the ap valve between the two chambers.
John Sutton MG, Tajik AJ, McGoon DC. Atrial septal defect in patients ages 60 years or older: operative results and long-term postoperative follow-up. Circulation 1981; 64: 4029. Kerut EK, Noreet WT, Plotnick GD, Giles TD. Patent foramen ovale: a review of associated conditions and the impact of physiological size. J Am Coll Cardiol 2001; 38: 61323. Park MK, Guntheroth WG. How to read pediatric ECGs, 4th edn. St Louis: Mosby, 2006.

Case 2 1. A, D The ECG shows sinus rhythm with no evidence of any type of heart block. There are widespread ST/T changes and an rSR pattern in the right chest leads, suggesting a partial bundle branch block. 2. A The widespread ST/T changes are suggestive of pericarditis. ECG changes for myocarditis can be very non-specic.

Pericarditis: pericardial inammation is commonly caused by viral or bacterial infections and immune-mediated disease (post-streptococcal immune complexes, connective tissue disease, and the presence of autoantibodies). Other causes include metabolic, neoplastic and parasitic causes. The ECG changes seen are due to small pericardial effusion and/or sub-epicardial pathology and are time dependant:

PAEDIATRICS AND CHILD HEALTH 17:5

213

SELF-ASSESSMENT

1. Acute changes: Widespread ST elevation, although this can be limited to the left ventricle (V56). 2. 23 days following insult: ST changes may normalise, and T wave may still be normal or slightly small. 3. 34 weeks following insult: T waves develop sharp inversion with iso-electric ST segment, which may persist for months. 4. Effusion may present with generalised low voltages. Myocarditis: myocarditis can occur at any age, including the newborn period, and may be related to a viral infection. Common aetiological pathogens include echovirus and coxsackie B virus. Symptoms vary greatly in severity and include fever, lethargy, breathlessness, chest pain, reduced exercise tolerance and palpitations. Infants may present with sweaty episodes and poor feeding. The ECG changes can often be non-specic and need to be correlated with the clinical picture: 1. Rhythm abnormalities (due to myocardial irritation) a. Unifocal/mulitifocal ventricular ectopics b. Varying patterns of AV conduction ranging form rst degree block through to a complete AV block c. Bundle or fascicular blocks 2. Prolonged QTc (corrected QT interval) 3. Decreased T-wave amplitude

Park MK, Guntheroth WG. How to read pediatric ECGs, 4th edn. St. Louis: Mosby, 2006.

Case 4 1. A, C, D The child has sinus rhythm; QRS axis is indeterminate as most of the limb leads are iso-electric. There is predominance of right ventricular forces dominant R in the right chest leads (V4R, and V1) with a prominent S wave in V6. Also, the presence of inverted T wave in V4R and V1 is highly indicative of right ventricular strain. 2. B The ejection systolic murmur along with the right ventricular hypertrophy and strain pattern suggest that the likely diagnosis is severe pulmonary stenosis. Right ventricular hypertrophy is seen in children with severe pulmonary stenosis, chronic lung disease and Eisenmenger syndrome. Large VSDs are likely to have features of biventricular hypertrophy rather than just RVH. In Ebstein anomaly, the atrial forces predominate, leading to peaked P waves with decreased RV forces. Patients with a dominant right ventricle and a small/ hypoplastic left ventricle are likely to have decreased left ventricular forces on the chest leads.

FURTHER READING
Brook MM. Cardiovascular system. In: Behrman RE, Kliegman RM (ed). Nelsons essentials of pediatrics, 4th edn. Philadelphia: WB Saunders, 2002. p. 58994. Park MK, Guntheroth WG. How to read pediatric ECGs, 4th edn. St. Louis: Mosby, 2006.

FURTHER READING
Emmanouilides GC (ed). Clinical synopsis. In: Moss and Adams: Heart disease in infants, children and adolescents. Philadelphia: Lippincott, Williams & Wilkins, 1998. Park MK, Guntheroth WG. How to read pediatric ECGs, 4th edn. St. Louis: Mosby, 2006.

Case 3 Consider the following when looking at ECGs 1. B, D 2. C The history of intermittent palpitations along with ECG changes (decreased PR interval and delta wave) is suggestive of WolfParkinsonWhite (WPW) syndrome. A cardio-memo recording revealed episodes of regular, narrow complex tachycardia, with heart rates of 200220/min. WPW syndrome occurs due to the presence of an accessory pathway between the atria and ventricles, with anterograde conduction (atria to ventricles). This pathway bypasses the normal AV node conduction delay and causes a pre-excitation which leads to premature ventricular depolarisation. Treatment is dependent on age, symptoms (frequency, duration, potential for serious sequelae) and patient choice. Treatment modalities range from no treatment, to medication such as beta-blockers and radio-frequency ablation.
 Rate: at a speed of 25 mm/s each small square is 0.04 s and

each big square is 0.2 s. In sinus rhythm, heart rate 300/ number of big squares.  Rhythm: a P wave should precede each QRS and there should be a regular PR interval. Normal p axis has upright p waves in I, II and aVF, though a normal variant will show inverted P waves in these leads, suggesting that the site of atrial activity originates from a lower point in the atrium, quite often the mouth of the coronary sinus.  QRS axis: best seen from I and aVF (Figure 5). Ventricular dominance: the axis varies with age of child. Right ventricular dominance at birth is followed by mixed right and left dominance followed by left ventricular dominance. At birth: +60 to +180 3 years: 0 to +110 16 years: +30 to +90 Superior axis is found in AVSD or tricuspid atresia.

FURTHER READING
Emmanouilides GC (ed). Clinical synopsis. In: Moss and Adams: Heart disease in infants, children and adolescents. Philadelphia: Lippincott, Williams & Wilkins, 1998.

PAEDIATRICS AND CHILD HEALTH 17:5

214

SELF-ASSESSMENT

90 120 aVR 150 60 aVL

3. q wave V1 4. Upright T in children older than 4 days, providing that the T is upright in V56.
30

Left ventricular hypertrophy: a combination of (age related): 1. QRS: a. R in I, II, II, aVL, aVF, V56 greater than upper limit of normal b. S in V12, V4R greater than upper limit of normal 2. R/S in V12 less than lower limit of normal Flattening or inversion of T in the left chest leads or I/aVF with concomitant ST changes would be suggestive of left ventricular strain. The presence of a delta wave is always associated with a short PR interval. T wave Positive in V1 in week 1, then negative until 10 years old, before becoming positive again. Negative T waves in V6 are always abnormal. They suggest myocarditis or cardiomyopathy: Flat: hypokalaemia, hypothyroidism; Peaked: hyperkalaemia, left ventricular overload, pericardits.

180

150

30

120 III 90 aVF

60 II

Figure 5 Pictorial representation of the frontal hexaxial reference system constructed by combination of two triaxial reference systems.
 P wave: best seen in the right chest leads and inferior leads.

Right atrial enlargement 42.5 mm (P pulmonale); left atrial enlargement 40.09 s duration (P mitrale).  PR interval: normal is 35 small squares (0.120.2 s). Decreased in WPW syndrome. Prolonged in heart block of any degree, ischaemia, hypokalaemia, Ebstein anomaly, rheumatic fever, digoxin therapy and myocarditis.  Q wave: normal in I, III and aVF, V5, V6. Should beo5 mm ando0.3 s  QRS complexes: normally up to 3 small squares (0.12 s).

QT interval This is measured from the beginning of the QRS complex to the end of T waves: QTc QT interval p RR

Bundle branch block Complete if QRS complex is wider than 0.12 s and partial if QRS complex is not prolonged. Left bundle branch block is very uncommon in children. The mnemonics Marrow and William are helpful: Right bundle branch block: M in lead V1 and W in V6 (i.e. MaRRoW). Left bundle branch block: W in lead V1 and M in V6 (i.e. WiLLiam). NB: You can comment on the axis with a partial bundle branch block but not with a complete bundle branch block. Ventricular hypertrophy A combination of various patterns need to recognise these changes as no one pattern can be considered to be diagnostic. In addition, always refer to the age-related ranges when interpreting paediatric ECGs, especially for abnormalities. Right ventricular hypertrophy: a combination of (in absence of right bundle branch block) (ranges for age): 1. Right axis dominance 2. Increased rightward ventricular forces: a. R/S ratio 4 1 in V12 b. Increased R in V1 or S in V6

Normal range 0.350.45 s (age-dependent). An abnormal QTc is prolonged: 1. 2. 3. 4. 5. Hypocalcaemia Myocarditis/diffuse myocardial involvement Cerebrovascular accident Long QT syndrome Drugs: macrolides, cisapride, tricyclic antidepressants

Acknowledgement
With special thanks to Dr O Stu mper and Mr J Stickley.

FURTHER READING
Emmanouilides GC (ed). Clinical synopsis of Moss and Adams heart disease in infants, children and adolescents. Philadelphia: Lippincott Williams & Wilkins, 1998. Keane JF, Flyer DC, Lock JE. Nadas Pediatric cardiology, 2nd edn. Philadelphia: WB Saunders, 2006. Park MK, Guntheroth WG. How to read pediatric ECGs, 4th edn. St. Louis: Mosby, 2006. Perloff JK. Clinical recognition of congenital heart disease, 5th edn. Philadelphia: WB Saunders, 2003. Rudolph AM. Congenital diseases of the heart: clinical physiological considerations, 2nd edn. Oxford: Blackwell Futura, 2001.

PAEDIATRICS AND CHILD HEALTH 17:5

215

SELF-ASSESSMENT

Self-assessment questions
Question 1
You are called to Accident and Emergency where a 7-year-old infant has been carried into the resuscitation area by his parents. Ventilation is being performed by bag and mask, and cardiac compressions are being performed. Compressions are stopped briey to assess the patient. A cardiac monitor shows the following trace (Figure 1). The infant has no palpable pulse and a manual debrillator is available. It has the following shock settings: 10, 20, 40, 50, 80, 100, 150, 200, 240 and 360 J. 1. What action do you take at this stage? (a) Continue cardiopulmonary resuscitation (CPR), this is a non-shockable rhythm (b) Give a 240 J shock (c) Give a 40 J shock, then check the rhythm and feel for a pulse (d) Give a 50 J shock, then check the rhythm and feel for a pulse (e) Give an 80 J shock, then check the rhythm and feel for a pulse (f) Give a 100 J shock, then check the rhythm and feel for a pulse (g) Give a 40 J shock, then continue CPR without reassessing the rhythm or checking the pulse (h) Give a 50 J shock, then continue CPR without reassessing the rhythm or checking the pulse (i) Give an 80 J shock, then continue CPR without reassessing the rhythm or checking the pulse (j) Give a 100 J shock, then continue CPR without reassessing the rhythm or checking the pulse (k) Give a 40, 40 80 J shock sequence, then check the rhythm and feel for a pulse (l) Give a 50, 50, 100 J shock sequence, then check the rhythm and feel for a pulse (m) Give a 40, 80, 80 J shock sequence, then check the rhythm and feel for a pulse (n) Give a 50, 100, 100 J shock sequence, then check the rhythm and feel for a pulse 2. Which of the following statements are true concerning advanced paediatric life support? (a) CPR should be performed at a rate of 5:1 in an infant and 15:2 in an older and larger child (b) The initial intravenous dose of adrenaline is 10 mg/kg (c) Subsequent intravenous doses of adrenaline are 100 mg/ kg
Christopher James, BSc (lond) BM BCh (oxon) is a Paediatric Specialist Registrar at the Gloucester Royal Hospital, Gloucester, UK. Celia Bagshaw, MB BS (birm) is an Accident and Emergency Specialist Registrar at the Gloucester Royal Hospital, Gloucester, UK.

(d) Once the trachea is intubated, cardiac compressions should be uninterrupted (e) The pulse should be checked for no longer than 15 seconds (f) If given down the tracheal tube, the dose of adrenaline should be 100 mg/kg (g) If intubated, chest compressions are given at a rate of 100 per minute and ventilation at a rate of 10 breaths per minute

Question 2
For each of the following sets of clinical features, pick the drug or poison most likely to cause the symptoms and signs listed if taken in overdose by a 5-year-old child. (a) (b) (c) (d) (e) (f) (g) Paracetamol Digoxin Deadly nightshade Atenolol Temazepam Carbon monoxide Holly berries

1. Headache, nausea, weakness, tachypnoea, skin blisters and rhadomyolysis 2. Vomiting, confusion, seizures, bradycardia and hypotension 3. Fever, sinus tachycardia, hypertension, ataxia, nystagmus and drowsiness

Question 3
A 5-year-old girl is brought to the Emergency Department with burns. She had been playing in the kitchen and had reached up and grabbed a saucepan from the stove. The pan contained vegetable oil that had been heated to a high enough temperature to fry potatoes for her dinner. After performing the primary survey and providing the child with analgesia, you decide to refer her to the regional burns centre. 1. Which two methods can be used to assess her percentage burns? (a) Assessment of all the areas of full-thickness burns. One percent surface area correlates to the area of the patients palm and the abducted ngers (b) Assessment of all the areas of partial- and full-thickness burns. One percent surface area correlates to the area of an adults palm and the adducted ngers (c) Assessment of all the areas of partial- and full-thickness burns. One percent surface area correlates to the area of the patients palm and the adducted ngers (d) Assessment of the percentage area using the rule of nines

PAEDIATRICS AND CHILD HEALTH 17:6

249

r 2007 Elsevier Ltd. All rights reserved.

SELF-ASSESSMENT

(e) Assessment of all the areas of partial- and full-thickness burns. One percent surface area correlates to the area of the patients palm excluding the ngers (f) Paediatric burns chart (g) Assessment of all the areas of partial- and full-thickness burns including erythema. One percent surface area correlates to the area of the patients palm with the ngers adducted 2. You assess the surface area as 20% and decide to commence her on intravenous uids. Assuming the child is a normal weight for her age, how much uid therapy would you give her in the rst 8 hours? (a) 1440 ml crystalloid in addition to her normal uid requirement (b) 480 ml crystalloid in addition to her normal uid requirement (c) 720 ml colloid in addition to her normal uid requirement (d) 1440 ml crystalloid that includes her normal uid requirement (e) 480 ml crystalloid that includes her normal uid requirement (f) 720 ml crystalloid in addition to her normal uid requirement (g) 480 ml packed red cells in addition to her normal uid requirement

Question 4
A 14-year-old girl is brought to the Emergency Department by her teacher. She was taking part in a cross country run and was seen to collapse to the ground. She lost conciousness for about 10 seconds and quickly returned to a normal clinical state. She has no past medical history of note and a physical examination is unremarkable. Her electrocardiogram (ECG) is shown below (Figure 2). 1. What abnormality does her ECG show? (a) Atrial brillation (b) Torsades de pointes (c) Long QT syndrome (d) WolffParkinsonWhite syndrome (e) Left ventricular hypertrophy 2. Which two statements regarding this condition are false? (a) This condition may be hereditary (b) It is a benign, self-limiting condition (c) The ECG changes may be drug related (d) The ECG changes may be associated with hypercalcaemia (e) The patient is at risk of developing ventricular brillation

Question 5
For the following questions, choose the best answer from the following list: (a) (b) (c) (d) (e) (f) (g) (h) Perthes disease Septic arthritis HenochSchonlein purpura Juvenile idiopathic arthritis Slipped upper femoral epiphysis Sickle crisis Transient synovitis (irritable hip) Joint involvement in haemophilia

Figure 1

1. A 4-year-old male presents to Accident and Emergency with a painless limp, present for the last week. On examination, there is reduced abduction and internal rotation 2. An 11-year-old boy presents to Accident and Emergency with a painful limp. All hip movements are painful bilaterally 3. A 41 2-year-old girl presents with a painless limp. She is usually well but has had a recent sore throat. She is playful in the department but appears to be in discomfort when you examine her left hip

Answer 1
1. j The rhythm strip shows ventricular brillation, a shockable rhythm. To calculate the joules required, the childs weight must be estimated using the formula (age+4) 2. In this case, the estimated weight is thus 22 kg. The 2005 resuscitation guidelines recommend giving one shock of 4 J per kilogram if

Figure 2

PAEDIATRICS AND CHILD HEALTH 17:6

250

r 2007 Elsevier Ltd. All rights reserved.

SELF-ASSESSMENT

using a manual debrillator, and then resuming CPR immediately without reassessing rhythm or feeling for a pulse. The debrillator present in this case gives the option of using 80 or 100 J, and we have calculated that our child requires 88 J. The resuscitation council recommend rounding up in all cases, and so this child should receive a 100 J shock with CPR resuming immediately afterwards for 2 minutes. After 2 minutes, the rhythm should be reassessed. There is no place for using an initial shock of 2 J/kg as recommended in the 2000 guidelines. 2. b, d, f and g The 2000 resuscitation guidelines suggested using a CPR ratio of 5:1 in small children and 15:2 in larger children. This often caused confusion in resuscitation situations and so the 2005 guidelines recommend using a ratio of 15:2 for all cases (except the newborn baby being resuscitated in the neonatal period where the ratio is 3:1). When giving adrenaline via the intravenous or intra-osseous route, a dose of 10 mg/kg is recommended for the rst and subsequent doses. This equates to 0.1 ml/kg of 1:10 000 solution. The higher dose of 100 mg/kg is only used if intravenous or intra-osseous access has proven difcult and adrenaline is to be given down the tracheal tube. Once the trachea is intubated, chest compression and ventilation should be continuous as long as this does not interfere with satisfactory ventilation. CPR should be interrupted only for debrillation. If intubated, the compression rate should be 100 per minute, and a ventilation rate of 10 per minute. When checking for a pulse, you should feel for the carotid pulse in a child, and the brachial pulse in an infant, for no longer than 10 seconds.

Toxicity is predominantly due to the anticholinergic effects and may include cardiac dysrhythmias and convulsions.

Answer 3
1. c and f 2. f The surface area of a burn can be assessed using a paediatric burns chart. An adult chart should not be used due to the differences in the surface areas of children and adults. In addition, the surface area of the patients palm with the ngers held adducted correlates to 1% of the total body surface area and is a useful tool at any age. Erythema should not be included when assessing the percentage burn. According to the APLS guidelines for the management of children with burns, intravenous uid therapy should be commenced in any child with 10% or more burnt skin. The Parkland formula for calculating the amount of crystalloid given per 24 hours is shown below, half of which is given over the rst 8 hours. This is in addition to the normal daily uid requirement. Percentage burn weight (kg) 4 Regional centres may advocate different uid therapies and these should be adhered to as appropriate.

Answer 4
1. c The ECG shows a prolonged QT syndrome. The QT interval is measured from the start of the QRS complex to the end of the T wave. The interval shortens with an increased heart rate, therefore to assess whether a patient has a prolonged QT interval, the corrected QT interval (QTc) is calculated using the formula below: QTc QT/ORR, QT represents the measured QT interval in seconds and RR is the measured RR interval. The normal QTc interval is 0.350.43 seconds long. This condition is clinically important as patients with long QT syndrome may suffer syncopal attacks, seizures, palpatations or sudden death secondary to ventricular tachycardia, torsade de pointes and ventricular brillation. These may be precipitated by exercise or fright. 2. b and d Long QT syndrome can be part of a congenital condition or the QT interval may be prolonged by acquired phenomena, for example, drug effects, metabolic disturbance or acute cardiac pathology. Hereditary syndromes include RomanoWard syndrome and JervellLangeNielsen syndrome, which may be associated with congenital deafness. Drugs that cause QT prolongation include quinidine, ecanide and tricyclic antidepressants. In addition, patients

Answer 2
1. f These are all immediate features of acute carbon monoxide poisoning. Chronic poisoning may also occur which may be more difcult to diagnose. Accidental carbon monoxide poisoning most commonly occurs when faulty heating systems are used in unventilated areas. Poisoning occurs by impairment of oxygen delivery as carbon monoxide binds to haemoglobin in preference to oxygen, thus resulting in cellular hypoxia. Treatment is predominantly supportive with oxygen being given in the highest concentration possible. 2. b Cardiac effects of digoxin toxicity may take up to 6 hours or more to develop. A profound bradycardia with prolonged PR and QRS occurs which may progress to atrioventicular block and sinus arrest. Central nervous system features such as confusion and convulsions may occur. Metabolic abnormalities include hyperkalaemia and a metabolic acidosis. 3. c The botanical name of Atropa Belladonna assists in remembering the toxic features from ingestion of this shrub.

PAEDIATRICS AND CHILD HEALTH 17:6

251

r 2007 Elsevier Ltd. All rights reserved.

SELF-ASSESSMENT

with hypocalcaemia, hypothermia or acute myocarditis may present with a prolonged QTc.

tall, thin children, usually boys aged 1015 years. It is bilateral in 25% of cases. The condition should be considered an emergency and requires epiphysiodesis for mild slips and possible open reduction and xation for bigger slips. 3. g Transient synovitis is the most common cause of limp in children, often occurring after an upper respiratory tract infection. A full blood count and erythrocyte sedimentation rate should be performed to rule out a septic arthritis if there is any clinical doubt. The condition usually improves within a couple of weeks of bed rest and analgesics.

Answer 5
1. a This child has Perthes disease, avascular necrosis of the femoral head. Peak incidence is between 4 and 10 years, with males being affected more than females. Presentation is usually with a limp, which can be painless or painful. Extensive involvement of the epiphysis may result in permanent femoral head deformity. Treatment consists of the principle of containment, often by abduction bracing and possibly osteotomy. 2. e This boy has bilateral slipped upper femoral epiphysis, involving a posterior separation of the femoral epiphysis from the metaphysis through the growth plate. It tends to occur in two distinct types of children, obese, hypogonadal children and

FURTHER READING

1 2

National Poisons Information Service. /www.toxbase.orgS Advanced paediatric life support manual, 4th ed. ALSG.

PAEDIATRICS AND CHILD HEALTH 17:6

252

r 2007 Elsevier Ltd. All rights reserved.

SELF-ASSESSMENT

Self-assessment Questions
Question 1
A 6-year-old boy is referred by his GP with a 6-month history of headaches. The headaches are generalised and occur characteristically in the morning. He does not report any particular precipitating or relieving factors. His mother is also concerned with his recent behaviour and thinks that he has a very poor attention span. She mentions that he is signicantly hyperactive compared with her other child and has started to wet his bed after having been dry since the age of 3 years. On further enquiry, she reports that he is developing normally and is otherwise healthy, though she thinks that he sometimes seems restless at night during sleep and she has heard him snore intermittently. On examination, he is healthy with height on the 75th centile and weight above the 91st centile for his age. His systems examination, including a detailed neurological examination, is normal except that he has mouth breathing and enlarged tonsils. Fundoscopy is normal.
(a) What is the most likely explanation for the symptoms? Intracranial space-occupying lesion Benign intracranial hypertension Attention-decit hyperactivity disorder Obstructive sleep apnoea Migraine Which one of the following tests is the most appropriate to investigate his symptoms? Cranial Computed Tomography Scan Pharyngolaryngoscopy Sleep study Referral to community paediatrician for management of attention-decit hyperactivity disorder Diagnostic lumbar puncture What management is most appropriate? Advise about weight reduction and referral to dietitian Adenotonsillectomy Neurosurgical referral Methylphenidate Lumbar puncture and acetazolamide

50th centile. Developmentally, he can respond to his mothers voice, achieved head control at 3 months of age and developed social smiling at 2 months. There are no concerns about feeding or swallowing food. In view of his normal development, his head size is monitored over the following 3 months. At 7 months, his head continues to be above the 97th centile and his weight is between the 50th and 75th centiles. Examination at this time reveals a large head with frontal bossing, a depressed nasal bridge and possibly coarse facial features. Examination of the chest reveals pectus excavatum and a thoracic gibbus. Cardiovascular, respiratory and abdominal examinations are normal. Neurological examination reveals normal tone and reexes. The anterior fontanelle is open and normal in size. Developmentally, he has not yet rolled over but is able to sit with support, though with a curved spine. The rest of the musculoskeletal examination is normal.
(a) What is the most likely clinical condition causing these features? Familial large head Mucopolysaccharidosis Hydrocephalus Skeletal dysplasia Canavan disease (b) Which basic investigation would provide a clue to the diagnosis? Computed Tomography Scan of the head Karyotyping Skeletal survey Enzyme assays in cultured broblasts Urine for glycosaminoglycans (c) Canavan disease is a type of Lysosomal storage disorder Lipid storage disorder Leukodystrophy Gangliosidosis Glycogen storage disorder

(b)

(c)

Question 3 Question 2
A 4-month-old boy is referred by his GP with a large head. He was a term baby born by normal vaginal delivery with no antenatal or postnatal concerns. He has a 5-year-old sibling who is t and well. The head size of both parents is on the
Krishna Jada MBBS MRCPCH is Specialist Registrar at the Department of Paediatrics, Watford General Hospital, Watford WD18 0HB, UK. Ashley Reece MBCHB MSC MRCPCH ILTM is Consultant Paediatrician at the Department of Paediatrics, Watford General Hospital, Watford WD18 0HB, UK. E-mail address: ashley.reece@nhs.net

A 3-week-old girl is brought by her mother to the Emergency Department with a history of vomiting for the last 4 days. The mother reports that the infant is vomiting after every feed and that the vomitus is white/yellow in colour. She has had three wet nappies in the last 24 hours and has opened her bowels twice. She was born by emergency Caesarean section for fetal bradycardia at 38 weeks and weighed 3.5 kg at birth. After birth, she was observed in a special care baby unit for 24 hours for grunting and being a bit mucousy. She was discharged home on day 3. All of her antenatal scans were reported as normal. She is bottle-fed, takes eight to nine bottle feeds per day and seems to be hungry most of the time. Her weight on assessment

PAEDIATRICS AND CHILD HEALTH 17:7

300

r 2007 Elsevier Ltd. All rights reserved.

SELF-ASSESSMENT

is 3.5 kg. On examination, she appears active and alert and is mildly jaundiced. Her vital signs are as follows.
Heart rate Respiratory rate Oxygen saturation Blood pressure Systems examination 165/min 40/min 97% in air 72/52 mmHg Normal

(a)

(b)

(c)

What is the most likely diagnosis? Gastro-esophageal reux disease Intussusception Malrotation Infantile hypertrophic pyloric stenosis Tracheo-esophageal stula What one investigation is most likely to help make a diagnosis? Abdominal ultrasonography Plain abdominal radiography Upper gastrointestinal contrast study pH study Tube oesophagography How is this condition treated? Nissen fundoplication Explorative lapratomy Ramstedts pyloromyotomy Closure of stula and end-to-end anastomosis Air enema

The simplest test for OSA is a sleep study comprising of overnight pulse oximetry. Abnormal oximetry can suggest a sleep-related breathing disorder; however, normal oximetry does not exclude OSA. Polysomnography (PSG) is considered the gold standard for the diagnosis of OSA, but it may not be required to make the diagnosis in all patients. Limited PSG measures the heart rate, respiratory effort, airow, movement, oximetry, and arterial pCO2. A more detailed PSG includes EEG, electro-occulography and submental electromyography to allow sleep staging and neurological arousals, but this is not universally available and is technically difcult. Children with Downs syndrome, craniofacial abnormalities, achondroplasia, mucopolysaccharidoses, PraderWilli syndrome and neuromuscular disease are more predisposed to OSA. In most of these conditions, adenotonsillectomy should be considered as the rst-choice treatment, though these children are at risk of incomplete resolution of the symptoms even after surgery. In children with no underlying cause, the most common reason for OSA is adenotonsillar hypertrophy, and hence adenotonsillectomy can resolve or signicantly improve the symptoms. Obesity is a risk factor for OSA and weight reduction should be part of the long-term treatment. Untreated OSA can have serious consequences such as learning and behavioural disorders, failure to thrive, pulmonary hypertension and cor pulmonale.

FURTHER READING
Primhak R, OBrien C. Sleep apnea. Arch Dis Child Educ Pract Ed 2005;90:ep87ep91. Rosen CL, Kass LJ, Haddad GG. Obstructive sleep apnea and hypoventilation. In: Behrman RE, Kliegman RM, Jenson HB, (eds.). Nelson textbook of paediatrics. Philadelphia: Saunders; 2004. p. 13971401.

Answers
Question 1 (a) Obstructive sleep apnoea (b) Sleep study (c) Adenotonsillectomy Obstructive sleep apnoea (OSA) occurs in children of all age groups with incidence of 0.73%. The incidence is highest in the pre-school age group, a period when the size of the adenotonsillar tissue is greatest in relationship to the airway size. The most common symptoms seen in children with OSA are snoring, gasps and witnessed apnoea during sleep, sleep disturbances, early morning headache, enuresis, mouth breathing, irritability, behavioural disorders and poor concentration. Daytime somnolence is a much less common symptom of OSA in children than in adults. The chronic nature of the headache, the absence of associated signs of raised intracranial pressure such as vomiting and nausea, and the normal fundoscopy and neurological examination make an intracranial space-occupying lesion and benign intracranial hypertension unlikely diagnoses. The nature of the headache is not characteristic of that seen in typical migraine. Children suspected to have attention-decit hyperactivity disorder should be assessed for OSA as a possible underlying cause, by history and clinical examination.

Question 2 (a) Mucopolysaccharidosis (b) Urine for glycosaminoglycans (c) Leucodystrophy Mucopolysaccharidosis (MPS) is a heterogeneous group of inheritable storage disorders caused by deciency of lysosomal enzymes leading to intralysosomal accumulation of glycosaminoglycans (previously called mucopolysaccharides). They are characterised by an initial period of apparent normality followed by appearance of various degrees of physical deformation, multi-organ involvement and mental retardation associated with excessive urinary excretion of mucopolysaccaharides (heparan sulphate, dermatan sulphate and keratan sulphate). Severe cases lead to premature death. There are several types of MPS, each type due to a specic enzyme deciency. All types of MPS are inherited in an autosomal-recessive manner except Hunter syndrome (MPS II), which is inherited as a X-linked-recessive condition.

PAEDIATRICS AND CHILD HEALTH 17:7

301

r 2007 Elsevier Ltd. All rights reserved.

SELF-ASSESSMENT

Classication of mucopolysaccaharidosis Type Eponym Enzyme deciency MPS I H Hurler a-L-Iduronidase syndrome

Important clinical manifestations Mental retardation, heart disease, coarse facies, dysostosis multiplex, hepatosplenomegaly MPS I S Scheie a-L-Iduronidase Normal intelligence, syndrome stiff joints and heart disease MPS I H/ HurlerScheie a-L-Iduronidase Intermediate between S Hurler and Scheie syndromes MPS II Hunter Iduronate Absence of corneal syndrome sulfatase clouding and peau d orange appearance of skin over scapula MPS III A Sanllippo Heparan N Physical features may syndrome A sulphatase be minimal and urine MPS III B Sanllippo N-acetyl-a-D screening marginally syndrome B glucosaminidase positive or negative; MPS III C Sanllippo Acetyl CoA: Nmay lead to undersyndrome C acetyl diagnosis of this glucosaminide condition N-acetyl Hyperactivity, transferase aggressive behaviour MPS III D Sanllippo N-Acetyl and profound mental syndrome D glucosamine-6- retardation sulphate sulphatase MPS IV A MorquioBrailsford Syndrome A N-Acetyl galactosamine- Skeletal 6-sulphatase abnormalities MPS IV B especially in weightbearing joints, odontoid hypoplasia, short trunk and stature, normal intelligence, nonimmune hydrops MorquioBrailsford syndrome B bGalacosidase MPS VI MarotauxLamy Arylsulphatase B syndrome (N-acetylgalactosaminyl-4sulphatse) Normal intelligence, increased intracranial pressure with optic atrophy and heart disease MPS VII Sly syndrome b-Glucuronidase Non-immune hydrops deciency fetalis, mental retardation, heart disease, dysostosis multiplex MPS IX Hyaluronidase Peri-articular soft tissue masses and short stature

The classications MPS V and MPS VIII are no longer used. Analysis of urine for mucopolysaccharides is the initial diagnostic test. Any individual who is suspected of an MPS disorder based on clinical features, radiographic results or urinary mucopolysaccharide screening tests should have a denitive diagnosis established by enzyme assay. Serum, leucocytes or cultured broblasts are used as the tissue source for measuring lysosomal enzymes. Familial large head is an unlikely answer due to the normal size of the heads of both parents. Hydrocephalus does not explain the other musculoskeletal features seen in this infant. Skeletal dysplasias present as disproportion and deformity in the neonatal period; however, some milder forms may have a delayed presentation. Canavan disease is a metabolic disease caused by deciency of the enzyme aspartoacylase leading to spongy degeneration of the white matter of the brain. It is one of the most common and severe forms of leukodystrophy. Infants usually appear normal at birth and until 36 months of age, when they develop macrocephaly, severe hypotonia, persistent head lag and delayed milestones.

FURTHER READING
Besley G T N, Buist N R M. Lysosomal storage disorders. In: McIntosh N, Helms PJ, Smyth RL, (eds.). Forfar and Arneils textbook of paediatrics. Edinburgh: Churchill Livingstone; 1998. p. 115671. Kliegman RM, Muenzer JL. Mucopolysaccharidoses. In: Behrman RE, Kliegman RM, Jenson HB, (eds.). Nelson textbook of paediatrics. Philadelphia: Saunders; 2004. p. 4826.

Question 3 (a) Infantile hypertrophic pyloric stenosis (b) Abdominal ultrasonography (c) Ramstedts pyloromyotomy Infantile hypertrophic pyloric stenosis (IHPS) presents with persistent non-bilious vomiting soon after every feed starting typically after 34 weeks of age in a hungry child. The vomiting may or may not be projectile. About 5% of infants with IHPS also have jaundice associated with decreased levels of glucuronyltransferase. Male infants are affected four times as commonly as female infants. Consequences of delayed diagnosis include failure to thrive, dehydration and hypokalaemic hypochloraemic metabolic alkalosis. Examination may reveal visible gastric peristalsis, and the diagnosis can be conrmed by a test feed in which an experienced examiner may palpate the pyloric mass as a rm movable mass located just above and to the right of the umbilicus. If no mass is palpable, ultrasonography of the abdomen can conrm the diagnosis in most patients. Criteria for the diagnosis include pyloric thickness greater than 4 mm or an overall pyloric length greater than 14 mm. Ultrasonography has a sensitivity of approximately 95%. An infant diagnosed with pyloric stenosis is rst stabilised by giving uid resuscitation with the aim of correcting the uid

PAEDIATRICS AND CHILD HEALTH 17:7

302

r 2007 Elsevier Ltd. All rights reserved.

SELF-ASSESSMENT

and electrolyte disturbance. Once the acidbase and electrolyte disturbance is corrected, the stenosis is treated by Ramstedts pyloromyotomy. Visible gastric peristalsis is occasionally seen in small, thin, normal infants without pyloric stenosis. Severe gastro-esophageal reux may also be confused with pyloric stenosis, which usually presents as effortless vomiting after feeds without the metabolic disturbance seen in IHPS. Intussusception usually presents at 39 months of age with a painful cry, drawing up of the knees and going pale. Vomiting is a common symptom seen in intussusception; the classic redcurrant jelly stools are seen in only about one-half of patients and is a late sign. Oesophageal atresia with distal tracheo-oesophageal stula is the most common type. Babies usually present with frothing at the mouth, choking or becoming cyanosed with feeds. Malrotation

presents in the rst 2 days of life with bile-stained vomiting and intolerance of feeds.

FURTHER READING
Fenton TR. Gastrointestinal problems and jaundice of the neonate. In: McIntosh M, Helms PJ, Smyth RL, (eds.). Forfar and Arneils textbook of paediatrics. Edinburgh: Churchill Livingstone; 1998. p. 20620. Mackinlay GA, Watson ACH. Surgical paediatrics. In: McIntosh M, Helms PJ, Smyth RL, (eds.). Forfar and Arneils textbook of paediatrics. Edinburgh: Churchill Livingstone; 1998. p. 1768800. Wyllie R. Pyloric stenosis and congenital anomalies of the stomach. In: Behrman RE, Kliegman RM, Jenson HB, (eds.). Nelson textbook of paediatrics. Philadelphia: Saunders; 2004. p. 122931.

PAEDIATRICS AND CHILD HEALTH 17:7

303

r 2007 Elsevier Ltd. All rights reserved.

SELF-ASSESSMENT

Self-assessment
Questions
Question 1 A 6-week-old boy is referred to the Emergency department by his GP with a history of loose stools since birth. He opens his bowel 810 times per day, and his stool has a watery consistency and does not contain any blood or mucus. His mother reports intermittent abdominal distension occurring at least twice per week that resolves spontaneously. His parents are of Middle Eastern origin. Antenatally, his mother was treated conservatively for pre-eclampsia, and antenatal scans showed polyhydramnios. The baby was born in good condition at 34 weeks gestation by normal vaginal delivery. He has been fully-breast fed since birth. At birth, his weight was 2.2kg and he currently weighs 3kg. The immediate post-natal period was uneventful. Mother and baby were discharged on day 3. In the rst few days at home, his mother sought advice from the midwife because of the frequent, loose stools, but was reassured that breast-fed babies often have loose stools. He was seen by his GP for his 6-week check and was referred because of the poor weight gain and persistent loose stools. On examination, the baby is found to be dehydrated, and assessment by the local paediatric service is arranged. Blood tests are taken and stool samples are obtained. His blood results are as follows (normal ranges in parentheses). Sodium Potassium Chloride Urea Creatinine Phosphate Albumin Triglyceride Cholesterol Corrected calcium Alanine aminotransferase -glutamyl transferase Haemoglobin WBC count Platelets Blood glucose pH pCO2 131mmol/L (137145mmol/L) 3.1mmol/L (3.65.0mmol/L) 91mmol/L (98106mmol/L) 6.3mmol/L (2.56.1mmol/L) 17mol/L (62106mol/L) 1.86mmol/L (1.252.10mmol/L) 37g/L (3550g/L) 1.83mmol/L (<2.3mmol/L) 2.23mmol/L (<5.0mmol/L) 2.8. mmol/L (2.102.55mmol/L) 107 U/L (052 U/L) 59 U/L (532 U/L) 10g/L (11.312.3g/dL) 9.0109/L (6.0017.50109/L) 483109/L (150400109/L) 5.5mmol/L (3.55.5mmol/L) 7.47 (7.357.45) 7.62kPa (3.65.5kPa) HCO3 Base excess 44.2mmol/L (2229mmol/L) 21mmol/L (4 +2mmol/L)

(a)  Which of the following best describes the above blood results? Hyponatraemic dehydration Acute renal failure Hyponatraemic hypokalaemic metabolic alkalosis  Hyponatraemic hypokalaemic compensated respiratory alkalosis  Hyponatraemic hypokalaemic hypochloraemic metabolic alkalosis (b)  The blood results are not compatible with which of the following conditions? Infantile hypertrophic pyloric stenosis Cystic brosis Allergic colitis of infancy Chloridorrhoea Bartters syndrome (c)  Which one of the following is most likely to provide a diagnosis? Abdominal ultrasonography Culture of stool Measurement of electrolyte content in the stool Colonoscopy and biopsy Therapeutic trail of exclusive amino acid formula milk Question 2 An 8-week-old girl is referred to the childrens emergency department with a history of passing blood in her stool for the last 7 days. Her mother says that she is otherwise well. Her stool is loose and is mixed with fresh, dark blood and occasionally contains mucus. There is no history of vomiting or fever. She is not in pain and is gaining weight along the 25th centile. She was born at term by normal delivery and was given an intramuscular dose of vitamin K at birth. She passed meconium on her rst day of life. She is exclusively breastfed and feeds every 34 hours. Her mother has asthma that is well controlled on an inhaled bronchodilator, which she takes as required; she is not taking any other medications. The girl has a 3-year-old brother who has severe eczema and who had recurrent wheezing episodes when he was an infant. (a)  What is the most likely diagnosis? Meckels diverticulum Intestinal polyp Intussusception Allergic colitis of infancy Haemorrhagic disease of the newborn (b)  What is the best advice for the management of this condition?  Allow the mother to continue breast-feeding but suggest that she excludes dairy products from her own diet

Babu Vadamalayan is Specialist Registrar at the Department of Paediatrics, Watford General Hospital, Watford, Hertfordshire WD18 0HB, UK. Ashley Reece is Consultant Paediatrician at the Department of Paediatrics, Watford General Hospital, Watford, Hertfordshire WD18 0HB, UK.

PAEDIATRICS AND CHILD HEALTH 17:8

340

2007 Elsevier Ltd. All rights reserved.

SELF-ASSESSMENT

 Advise her to stop breast-feeding and give formula feed derived from cows milk  Advise her to stop breast-feeding and give a pre-digested cows milk formula  Advise her to stop breast-feeding and give soya milk-based formula  Advise her to continue breast-feeding but to begin early weaning to solids Question 3 An 11-year-old boy is referred with history of recurrent abdominal pain for the last 4 months. According to his parents, he has been suffering heartburn since he was small. He complains of pain mainly around the epigastric region. At the onset of the symptoms, he was seen by his GP, who prescribed Gaviscon. This relieved his pain to some extent, but his symptoms persisted. At the same time, he was also treated for constipation with lactulose, but this did not make any difference to his abdominal pain. He describes his pain as dull, typically occurring 23 hours after a meal and occasionally waking him from sleep. Recently, the episodes have have increased in frequency. He also complains of a loss of appetite, but there is no history of weight loss. He is nauseous, but there was no history of vomiting. His school reports have been good and there are no worries or stresses at home. He lives with his parents and his older brother. They have two cats. He has not been taking any non-steroidal anti-inammatory drugs. The previous month, he was seen in the emergency department and diagnosed with gastritis. He was treated with oral omeprazole without any improvement in his symptoms. On examination, the only positive nding is epigastric tenderness. His height and weight are on the 50th centile. His blood count, ESR, C-reactive protein, liver function, amylase and coeliac screening (tissue transglutaminase antibodies) are all normal. He undergoes oesophagogastroduodenoscopy under general anaesthesia. This reveals multiple ulcers and nodularity in the gastric body, antrum and duodenum (Figure 1). A biopsy was taken and sent to the laboratory for microscopy.

(a)  What is the most likely diagnosis? Gastric Crohns disease ShwachmanDiamond syndrome Gastritis due to Helicobacter pylori infection Abdominal migraine Lactose intolerance (b)  What tests can be done for a rapid bedside diagnosis at the time of endoscopy? Urea breath test Hydrogen breath test Camplyobacter-like organism test PCR for Helicobacter pylori on gastric mucosal biopsy Stool antigen test for specic microbes (c)  How do you treat this condition?  Prokinetic agent and proton pump inhibitor for 2 weeks Oral penicillin V and oral metronidazole for 2 weeks  Oral amoxicillin, oral clarithromycin and proton pump inhibitor for 1 week  Oral amoxicillin, oral clarithromycin and probiotics for 6 weeks  Gaviscon, oral amoxicillin and oral ciprooxacin for 1 week

Answers
Answer 1 (a)  Hyponatraemic hypokalemic hypochloraemic metabolic alkalosis (b)  Allergic colitis of infancy (c)  Measurement of electrolyte content in the stool This baby has features of congenital chloridorrhoea. This condition is also known as congenital alkalosis with diarrhoea or DarrowGamble syndrome. It is an autosomal recessive condition caused by defective ileal and colonic ClHCO3 exchange. It is prevalent in Finland (1/20,000), Saudi Arabia, Poland and Kuwait. Faecal chloride levels greater than 90mmol/litre or chloride levels greater than sodium plus potassium are characteristic. Antenatal diagnosis is possible by DNA analysis. At least 30 different mutations involving the SLC26A3 gene (solute carrier gene 3) are described. Congenital chloridorrhoea is associated with polyhydramnios and consequently premature birth. It is lethal if not treated. Complications are mainly due to chronic dehydration. Lifelong chloride and sodium supplements are necessary. Bartter syndrome causes hypokalemic alkalosis and hypercalciuria. This is a hereditary condition associated with renal tubular defect. Pyloric stenosis, cystic brosis and chloridorrhoea also causes similar electrolyte abnormalities but not hypercalciuria (otherwise called pseudo Bartters syndrome).

Figure 1 Findings at upper gastrointestinal endoscopy.

PAEDIATRICS AND CHILD HEALTH 17:8

341

2007 Elsevier Ltd. All rights reserved.

SELF-ASSESSMENT

FURTHER REaDING Hihnala S, Hoglund P, Lammi L, et al. Long-term clinical outcome in patients with congenital chloride diarrhoea. J Pediatr Gastroenterol Nutr 2006; 42: 36975. Li WC, Shih HH, Wu KL, et al. Congenital chloride diarrhoea in a child. J Formos Med Assoc 2003; 102: 4248.

Answer 2 (a)  Allergic colitis of infancy (b)  Allow the mother to continue breast-feeding but suggest that she excludes dairy products from her own diet Allergic colitis of infancy is a major cause of rectal bleeding in infants. It presents as fresh blood passed per rectum in the rst few weeks to months after birth in an otherwise well baby. This is usually due to cows milk protein allergy. Rectal biopsy shows increased eosinophilic inltrate. Exclusion of cows milk products from the maternal diet in breast-fed infants or changing to hydrolysed (pre-digested) formula in bottle-fed infants is necessary. Cows milk protein intolerance often resolves by the age of 12 months. The current recommendation for weaning is to begin at 6 months. Soya milk is not advised in infants under the age of 6 months; 30% of children with cows milk protein allergy can also react to soya protein.

Helicobacter pylori is a Gram-negative, spiral-shaped bacterium that colonizes the stomach and causes silent gastritis. A small percentage of children are symptomatic with gastric and duodenal ulcers. Eradication of H. pylori is necessary in patients with a gastroduodenal ulcer associated with H. pylori gastritis. Eradication modies the natural history of the disease and greatly reduces the risk of recurrence. Serology for H. pylori antibodies is not specic. Examination of stool for H. pylori antigen and urea breath tests can be used, but endoscopy and biopsy are conrmatory. The Campylobacter-like organism test is conducted at the time of endoscopy. This bedside test is performed on a mucosal biopsy specimen that is inoculated into a medium containing urea and phenol red, a dye that turns pink at pH 6.0 or greater. The pH of the medium rises above 6.0 when H. pylori, a Campylobacter-like organism, metabolizes urea to ammonia by way of its urease activity; hence, a red colour indicates that the test is positive. This test is highly sensitive and specic. Triple therapy (comprising amoxicillin, clarithromycin or metronidazole and omeprazole) is effective in eradicating H. pylori infection; 1 week of treatment is sufcient. Treatment failure usually indicates antibacterial resistance or poor compliance.

FURTHER REaDING Anveden-Hertzberg L, Finkel Y, Sandstedt B, et al. Proctocolitis in exclusively breast-fed infants. Eur J Pediatr 1996; 155: 4647. Sorea S, Dabadie A, Bridoux-Henno L, et al. Hemorrhagic colitis in exclusively breast-fed infants. Arch Pediatr 2003; 10: 7725.

FURTHER REaDING Bittencourt PF, Rocha GA, Penna FJ. Gastroduodenal peptic ulcer and Helicobacter pylori infection in children and adolescents. J Pediatr (Rio J) 2006; 82: 32534. (Epub 2006 Sep 19). Delaney BC, Moayyedi P, Forman D. Initial management strategies for dyspepsia (Cochrane review). The Cochrane Library. Issue 2. Chichester: Wiley; 2003. Dondi E, Rapa A. High accuracy of non invasive tests to diagnose Helicobacter pylori infection in very young children. J Pediatr 2006; 149: 81721. Paediatric Formulary Committee. British national formulary for children. London: BMJ Publishing Group; 2006.

Answer 3 (a)  Gastritis due to Helicobacter pylori infection (b)  Campylobacter-like organism test (c)  Oral amoxicillin, oral clarithromcin and proton pump inhibitor for 1 week

PAEDIATRICS AND CHILD HEALTH 17:8

342

2007 Elsevier Ltd. All rights reserved.

SELF-ASSESSMENT

Self-assessment
Questions
Question 1 A 12-year-old boy is referred urgently by his general practitioner to the paediatric outpatient clinic. He has a 3-month history of non-specic illness after returning from a holiday in America. His main symptoms are lethargy, anorexia and weight loss, along with generalized joint aches. His appetite is decreased and he nds eating an effort. He has no signicant medical history. There is strong family history of systemic lupus erythematosis (SLE). On examination, his weight is on the 9th centile and his height on the 50th centile. He looks tired and has a purplishred rash around both eyes and across the nasal bridge. He also has a rash on the extensor aspects of his elbows and ngers. His speech is nasal in quality and there is a distinct lack of clarity in his words. Neurological examination reveals reduced power in his lower limbs (worse proximally) with normal tone, coordination and reexes. When asked to stand from sitting on the oor, he exhibits a positive Gowers sign. Respiratory, cardiovascular and gastrointestinal examinations are all normal. (a)  What is the likely diagnosis? SLE Juvenile dermatomyositis Post-viral fatigue syndrome Becker muscular dystrophy Anorexia nervosa (b)  Which investigation would be least helpful in establishing the diagnosis? ESR and C-reactive protein (CRP) Autoantibody screening Viral serology MRI with muscle biopsy and electromyography Creatine kinase (c)  What treatment would you like to commence? Calcium and vitamins Cognitive behavioural therapy Corticosteroids methotrexate Total parenteral nutrition Anticoagulants Question 2 An 11-year-old boy is referred from the dental hospital due to delayed eruption of his adult dentition. His only medical history is a squint and surgical orchidopexy. On examination, his weight is on the 50th centile and his height between the 2nd and 9th centile. He has mild frontal bossing with a prominent jaw and a wide nasal bridge. On examination of the mouth, only two of his lower central incisors are erupting; the remainder of his teeth are his primary dentition. He has a square-shaped ribcage. When asked, he is able to bring both shoulders around so that they meet anteriorly in the midline. The radiograph in Figure 1 shows the classical abnormality associated with this disease. (a)  What does the radiograph show? Cervical rib Osteopenia Osteopetrosis Clavicular hypoplasia Abnormal scapulae (b)  What is your diagnosis? Campomelic dysplasia Chondroectodermal dysplasia Pyknodysostosis Achondroplasia Cleidocranial dysostosis (c)  What is the usual inheritance pattern of this disease? Autosomal recessive Sporadic Autosomal dominant X-linked dominant X-linked recessive (d)  To which of the following specialties might you refer him for further input?

R Tindale MBChB is SHO Paediatrics, Department of Paediatrics, Queens Hospital, Belvedere Road, Burton-on-Trent, Staffordshire DE13 0RB, UK. M Ahmed MBBS FCPS MRCP FRCPCH is Consultant Paediatrician, Department of Paediatrics, Queens Hospital, Belvedere Road, Burton-on-Trent, Staffordshire DE13 0RB, UK. L Jones MBChB is FY1, Department of Paediatrics, Queens Hospital, Belvedere Road, Burton-on-Trent, Staffordshire DE13 0RB, UK.

Figure 1 Radiograph of upper chest (right half ).

PAEDIATRICS AND CHILD HEALTH 17:9

371

2007 Elsevier Ltd. All rights reserved.

SELF-ASSESSMENT

Orthopaedics Physiotherapy Respiratory Genetics Maxillofacial

Question 3 A boy aged 2 years 9 months presents to the emergency department with sudden onset of pallor and unresponsiveness. The history given by the parents is of an illness associated with cough, sore throat and vomiting for the last 2 days. His birth history is unremarkable and he has no history of jaundice or similar illness. He has an older brother and the family has only recently moved to the UK from Iraq. The paediatric team are crash bleeped and on examination nd him to be very pale and tachycardic (170 bpm). His respiratory rate is 40/minute, his temperature 37.4C and his blood pressure 101/41 mmHg. His capillary rell time is 3 seconds centrally. There is no jaundice, visible skin rash, lymphadenopathy or signs of chronic liver disease. He has splenomegaly of 5 cm. His liver is not enlarged. He responds to voice. His pupils are of normal size, and equal and reactive to light. There is no skin rash and no neck stiffness. Initial tests reveal a haemoglobin level of 2 g/dl with a retic ulocyte count of 9.1%. The WBC count is 15.6109/litre and the differential count is normal. The platelet count is 53109/litre. (a)  What is the most likely diagnosis? Bantis syndrome Chronic myeloid leukaemia (CML) Sickle cell crisis Malaria Typhoid fever (b)  What long-term treatment would need to be prescribed? Penicillin Iron supplementation Erythropoietin Vitamin B12 supplementation Non-steroidal anti-inammatory drug (c)  Which vaccination is useful in preventing complications in future? MMR Pneumococcal Varicella Hepatitis B Inuenza (d)  What is the inheritance pattern of this disease? Autosomal recessive Multifactorial Autosomal dominant X-linked dominant X-linked recessive Question 4 A 1-year old girl is being followed-up in the outpatient department when she is noted to have severe faltering growth and

gross motor developmental delay after initially feeding well with a bottle. She was born prematurely at 35 weeks by elective caesarean section due to intrauterine growth retardation and breech presentation. Her birthweight was 1580 g, on the 2nd centile, and her head circumference was 31 cm, on the 25th centile. There were no problems at delivery and she had a good Apgar score. There is no family history of signicance. She is extensively investigated for feeding difculties, vomiting and faltering growth, including renal, liver and thyroid function tests, coeliac serology, urine-reducing substances, faecal elastase, sweat test, urine amino and organic acids, full blood count, karyotyping, ultrasonogaphy of the kidneys and barium meal examination. All are normal and negative. To maintain a stable weight, she requires a nasogastric tube for top-up feeds. Because of subtle dysmorphic features, she is referred to the clinical geneticist, who notes that she sweats a lot around the head. She is now 14 months of age and has a large anterior fontanel with frontal bossing, relatively large eyes and a slightly triangular-shaped face with sharp features. She has no asymmetry of face or limbs but has bilateral fth nger clinodactyly. She shows some delay in gross motor development but is mentally alert. Her length is 3 cm less than the 0.4th centile and her weight is about 2 kg less than the 0.4th centile. Her head circumference is around the 50th centile (Figure 2). (a)  What is the likely diagnosis? Constitutional short stature Growth hormone deciency Turners syndrome RussellSilver syndrome Gastro-oesophageal reux disease (GORD) (b)  What chromosome abnormality is known to be associated with 10% of cases? 45X  Maternal uniparental disomy of chromosome 7 (UPD maternal 7) 5p Robertsonian translocation Maternal uniparental disomy of chromosome 15 (c)  What is the probability that the parents will have another child with the same disorder? 10% 33% 2% 7% 50% Question 5 From the age of 4 months, a boy has been repeatedly admitted to hospital for recurrent gastroenteritis and upper respiratory tract infections and has been extensively investigated for faltering growth and anaemia. He is of very small size, being below the 0.4th centile for both weight and height. He has thin muscle mass and experiences problems with poor

PAEDIATRICS AND CHILD HEALTH 17:9

372

2007 Elsevier Ltd. All rights reserved.

SELF-ASSESSMENT

91st 75th 50th 25th 9th 2nd 0.4th

48 47 46 45 44 43 42

84 82 99.6th 98th 91st 75th 50th 25th 9th 2nd 0.4th 80 78 76 74 72 70 68 66 64 62 60 58 56 54 52 Length (cm)

13 99.6th 12 98th 11 91st 75th 50th 9 25th 9th 2nd 0.4th 7 8 10

Usable with 95% & 5% thrive lines A Breast from Birth chart is available based on long-term breastfeeding data
EDD EDD EDD EDD EDD EDD EDD 1 38 2 4 6 8 10 12 14 16 18 20 22 24 26 28 30 32 34 36 38 40 42 44 48 50 52 3

Figure 2 Growth chart showing measurements during rst year of life.

PAEDIATRICS AND CHILD HEALTH 17:9

373

2007 Elsevier Ltd. All rights reserved.

SELF-ASSESSMENT

feeding. His skin is dry and scaly, especially over the torso. His bowel habits are normal, though his stool are occasionally smelly and greasy. His teeth are in poor condition; many are broken and there is severe dental caries. The family and school report a lack of concentration and some falling behind at school by the age of 5 years. There is a medical history of B thalassaemia trait, with a family history of thalassaemia trait and asthma. He was born at term by normal vaginal delivery without complications during the pregnancy or post-natal period. Various investigations have been performed over a period of time; these initially revealed positive gliadin antibodies on screening for coeliac disease, but duodenal biopsy at 1 year of age was normal. Careful review of the investigations revealed intermittent episodes of neutropenia and raised alanine aminotransferase. Further investigations show that his faecal elastase is low at 54 g/g (normal >200 g/g). Renal, liver and thyroid function tests and amylase are normal. A repeat coeliac screen and immunoglobulin screening are also normal. No fat globules are seen in a stool specimen. (a)  What is the most likely diagnosis in view of the above investigation results? Cystic brosis Pearsons syndrome Coeliac disease ShwachmanDiamond syndrome (SDS) Acquired immune deciency syndrome (AIDS) (b)  What treatment should be started? Pancreatic supplements and fat-soluble vitamins Antiretroviral therapy Gluten-free diet Cows milk protein-free and soya-free diet Broad-spectrum intravenous antibiotics for 2 weeks (c)  What haematological problems can these children encounter, necessitating regular follow-up by the paediatric haematology team? Megaloblastic anaemia  Premalignant and malignant haematological disorders (myelodysplasia or leukaemic transformation) Aplastic anaemia Iron-deciency anaemia Persistent neutropenia

Answers
Answer 1 (a)  Juvenile dermatomyositis (b)  Viral serology (c)  Corticosteroids methotrexate Dermatomyositis was considered as the likely diagnosis because of the presence of characteristic dermatological ndings along with proximal muscle weakness. Becker muscular

dystrophy presents in late adolescence or early adult life with muscle weakness, calf pseudohypertrophy, cardiomyo pathy, and elevated serum levels of creatine phosphokinase. Although SLE is a multisystem disease, it does not present with proximal muscle weakness or myositis. Similarly, in post-viral fatigue syndrome and anorexia nervosa, one does not expect these typical skin features or signs of proximal muscle weakness. Dermatomyositis is an idiopathic inammatory myopathy that often presents with constitutional symptoms such as fatigue, low-grade fever, weight loss and irritability, along with a characteristic skin rash and muscle weakness (mainly proximal). Derangement of upper airway function can be detected by hoarseness, a nasal quality to the speech, or difculty in handling secretions. Dysphagia is a severe prognostic sign and should prompt immediate and aggressive therapeutic intervention. Periorbital violaceous (heliotropic) erythema (heliotrope eyelids) is a characteristic but not pathognomonic sign in this condition. Malar erythema, poikiloderma in a photosensitive distribution, erythema on the extensor surfaces and hypertrophic, reddish-pink skin over the metacarpal and proximal interphalangeal joints (Gottrons papules) during active disease are also characteristic cutaneous manifestations of the disease. Muscle involvement manifests as proximal muscle weakness, with patients having difculty climbing stairs, rising from chairs and reaching for objects above their head. Muscle tenderness may also be a feature. The child cannot rise unassisted from the oor without climbing up the body (Gower sign). Inammatory markers (e.g. ESR, CPR) may be raised, and autoantibodies (especially antinuclear antibody) are often positive but not diagnostic of the disease. Muscle enzymes are often abnormal, the most sensitive being creatine kinase. MRI is useful in detecting muscle oedema and distinguishing corticosteroid myopathy from continued inammation. It is also useful in selecting a site for muscle biopsy if required. Treatment comprises of oral corticosteroids (intravenous methylprednisolone is often used in active disease) along with appropriate use of sunscreens. Other immunosuppressive therapies (e.g. methotrexate) are added to induce clinical and biochemical remission. Physiotherapy, occupational therapy and psychotherapy may be required during periods of exacerbation. The duration of active symptoms has decreased and the prognosis of this illness has improved with more aggressive immunosuppressive therapy. Answer 2 (a)  Clavicular hypoplasia (b)  Cleidocranial dysostosis (c)  Autosomal dominant (d)  Genetics The diagnosis of cleidocranial dysostosis was made because of the typical clinical and radiological features in

PAEDIATRICS AND CHILD HEALTH 17:9

374

2007 Elsevier Ltd. All rights reserved.

SELF-ASSESSMENT

this patient. Campomelic dysplasia presents in the newborn period with bowing of long bones (especially in the lower legs), respiratory distress and other anomalies including defects of the cervical spine and hypoplasia of the scapulae and pelvic bones. Chondroectodermal dysplasia (Ellisvan Creveld syndrome) is a skeletal and an ectodermal dysplasia that presents at birth with short limbs and polydactyly of the hands and sometimes of the feet. Pyknodysostosis presents in early childhood with short limbs, an open anterior fontanelle, a large skull with frontal and occipital bossing, and dental abnormalities. Achondroplasia typically presents at birth with short limbs (greatest in the proximal segments), a long narrow trunk, and a large head with mid-facial hypoplasia and a prominent forehead. Cleidocranial dysostosis is a congenital condition with autosomal dominant inheritance. The disorder involves abnormal development of the bones in the skull and the clavicle area. Patients with cleidocranial dysostosis have frontal bossing, a prominent jaw and a widened nasal bridge. The clavicles can be missing (clavicular agenesis) or abnormally developed (clavicular hypoplasia). As a consequence, affected individuals are able to bring their shoulders together anteriorly. They may also present with a large anterior fontanel, delayed ossication of the cranial bones with multiple ossication centres (wormian bones), and delayed ossication of pelvic bones. Tooth eruption is often delayed. In addition, the primary teeth can be abnormally retained while the permanent teeth remain unerupted. Commonly, the adult teeth develop late, and supernumerary teeth may cause the normal teeth to become crooked. There is usually a family history of the disease, though in this case neither parent was affected; the child had a new mutation. Referral to a geneticist for counselling is required when the patient wishes to start a family. The prognosis is usually good except for dislocations, especially of the shoulders, and dental anomalies, which require regular and intensive input from a dentist. Answer 3 (a)  Sickle cell crisis (b)  Penicillin (c)  Pneumococcal (d)  Autosomal recessive A sequestration crisis secondary to sickle cell disease was the likely cause of the sudden presentation in this patient (sudden onset with severe anaemia, splenomegaly and high reticulocyte count). Further tests, including a blood lm and haemoglobin electrophoresis, conrmed sickle cell disease. Parvovirus antibodies were negative. Patients with malaria and typhoid fever present with continuous or intermittent pyrexia. CML presents with an initial chronic phase in which the malignant clone produces an elevated leucocyte count with a predominance of mature forms but with

increased numbers of immature granulocytes. The spleen is often greatly enlarged, often resulting in pain in the left upper quadrant of the abdomen. Bantis syndrome (congestive splenomegaly) results from obstruction in the hepatic, portal or splenic veins. In neonates, this is usually secondary to congenital anomalies or infections, and it may result in portal hypertension in some cases. Sickle cell disease is an inherited disorder of haemoglobin synthesis that results in normocytic haemolytic anaemia. The abnormally shaped RBCs are susceptible to assuming a sickle shape when under stress. These sickle cells produce thrombosis and obstruction of the small vessels and thereby tissue ischaemia and necrosis distally. The genetic change is a single amino acid substitution of valine for glutamate at position 6 of the -globin chain of haemoglobin. This substitution leads to unstable RBCs with a reduced lifespan. The disease is a chronic illness with multiple acute exacerbations that can be life-threatening. The most common causes of death in children are infection (age 13 years) and stroke (age 1020 years). Other children have signicant morbidity and are at risk of mortality from acute chest syndrome, splenic sequestration crisis, parvovirus B19 infection and aplastic crisis, and vaso-occlusive crisis. The mainstay of treatment during a crisis (after airway, breathing and circulation) is adequate hydration and analgesia. Antibiotics should be given to cover infections with pneumococci, Haemophilus inuenzae type B, Salmonella and Mycoplasma pneumoniae. Oxygen and respiratory support should be given to maximize the oxygen-carrying capacity of the haemoglobin. In moderate-to-severe respiratory distress, blood transfusions should be given to increase the haemoglobin level. New therapies offer patients with sickle cell disease a prolonged lifespan, with the potential to prevent or minimize complications that impair quality of life. Advances in clinical therapy include prophylactic antibiotics, RBC pheresis, hydroxyurea, drugs to increase fetal haemoglobin concentration, including butyrate and decitabine, and angiotensinconverting enzyme inhibitors to prevent renal disease. Bone marrow transplantation has become accepted therapy in severely affected children. Answer 4 (a)  RussellSilver syndrome (b)  Maternal uniparental disomy of chromosome 7 (UPD maternal 7) (c)  In most families, when the parents are both of normal stature, it is unlikely that they will have another child with RussellSilver syndrome the risk is no more than 23% RussellSilver syndrome was diagnosed in this patient in view of the dysmortphic features along with the intrauterine/ post-natal growth retardation and was later conrmed on chromosome testing. GORD was an unlikely possibility because

PAEDIATRICS AND CHILD HEALTH 17:9

375

2007 Elsevier Ltd. All rights reserved.

SELF-ASSESSMENT

of the failure to gain weight even with tube-feeding (and the absence of vomiting). Constitutional growth delay is a variant of normal growth in which the length and weight of affected children are normal at birth and growth is normal for the rst 412 months of life. Isolated growth hormone deciency results in symmetrical growth delay without the dysmorphic features described in this case. Turners syndrome comprises sexual infantilism, a webbed neck and cubitus valgus in females. RussellSilver syndrome has an estimated incidence of 1/3000 to 1/50,000 live births. There are four main clinical components to the disease: characteristic facies, low birth weight, asymmetry and growth retardation. The cause remains unknown in most cases, but a few have a chromosome abnormality, most commonly in chromosome 7 or 17. In 10% of cases, RussellSilver syndrome Is due to UPD maternal 7. The primary abnormality is growth failure. Patients present early with intrauterine growth retardation, post-natal faltering growth and feeding difculties. Dysmorphic facial features are normally seen, with a triangular face, blue sclerae, micrognathia and a prominent nasal bridge. Most patients show normal intellectual development, though gross motor development may be delayed. The diagnosis is clinical and can be further conrmed in some cases by chromosomal analysis looking for UPD maternal 7 (blood samples should be sent from both the patient and the parents). Management is multifaceted and multidisciplinary, involving the feeding team/dietitian, physiotherapist, geneticist and community paediatrics (child development team). Referral to a paediatric endocrinologist should be considered, as treatment with growth hormone has been used to improve the nal height in some of these patients with documented growth hormone deciency. Answer 5 (a)  ShwachmanDiamond syndrome (SDS) (b)  Pancreatic supplements and fat-soluble vitamins (c)  Premalignant and malignant haematological disorders (myelodysplasia or leukaemic transformation) Recurrent infections, pancreatic insufciency, intermittent neutropaenia, faltering growth, short stature and poor dental health suggest the diagnosis of SDS. Although pancreatic insufciency and faltering growth can be features of cystic brosis, intermittent neutropenia is not a common nding. Moreover, these children present with predominant respiratory symptoms. Pearsons syndrome (another disorder of the exocrine pancreas) presents with pancreatic insufciency, faltering growth, variable thrombocytopenia and macrocytic anaemia. Antigliadin antibodies have poor sensitivity and specicity in the diagnosis of coeliac disease and have been replaced by tissue transglutaminase and endomysial antibodies. A negative duodenal biopsy (the gold standard) would exclude coeliac disease. AIDS has a variable presentation in infants and young children, with lymphadenopathy,

epatosplenomegaly, interstitial pneumonia, oral thrush h (usually persistent), faltering growth, chronic parotid swelling and recurrent bacterial infections. SDS is a triad of exocrine pancreatic insufciency, bone marrow failure and bony changes. After cystic brosis, it is the second most common cause of pancreatic insufciency in children and should always be considered when cystic brosis-like symptoms are present. In SDS, there is a considerable loss of pancreatic acini with fatty replacement, but duct and islet structure is preserved. Steatorrhoea is often noticed in the rst 6 months of life and in 90% of patients by 1 year. However, unlike in cystic brosis, pancreatic function usually improves with age. The predominant haematological feature is neutropenia, seen in more than 80% of patients. This can be persistent or intermittently severe with at levels of less than <0.5109/litre. Infants with SDS are proportionally small and grow along the lower centiles. The main skeletal defects are related to abnormal development of the growth plates, particularly the metaphyses. Metaphyseal chondrodysplasia occurs in 4080% of patients. The genetic abnormality in SDS was identied in 2002. More than 90% of patients have the alteration, in a single gene on chromosome 7. This abnormality was conrmed in this patient. Management of children with SDS should be multidisciplinary and include a general paediatrician, a paediatric gastroenterologist and haematologist, and a dentist, dietitian and psychologist. The mainstay of treatment is pancreatic enzyme supplements together with a high-calorie diet. The requirement for enzyme supplements may decrease with age, as studies have shown that almost 50% of SDS patients have a near-normal fat balance by 4 years of age. Depending on the degree of baseline haematological abnormalities, a full blood count is performed every 36 months, or more frequently if required by the symptoms. Annual or biennial surveillance bone marrow examinations are performed to look for cyto genetic abnormalities. SDS-related leukaemia has a poor prognosis. The projected median survival of patients with SDS has been calculated to be more than 35 years.

FURTHER REaDING Angle AD, Rebellato J. Dental team management for a patient with cleidocranial dysostosis. Am J Orthod Dentofacial Orthop 2005; 128: 11017. Boocock GRB, Morrison JA, Popvic M, et al. Mutations in SBDS are associated with Shwachman-Diamond syndrome. Nat Genet 2003; 33: 97101. Callen JP. Dermatomyositis: diagnosis, evaluation and management. Minerva Med 2002; 93: 15767. Claster S, Vichinsky EP. Managing sickle cell disease. BMJ 2003; 327: 11515. Hall GW, Dale P, Dodge JA. Shwachman-Diamond syndrome: UK perspective. Arch Dis Child 2006; 91: 5214.

PAEDIATRICS AND CHILD HEALTH 17:9

376

2007 Elsevier Ltd. All rights reserved.

SELF-ASSESSMENT

Hitchins MP, Stanier P, Preece MA, et al. Silver-Russell syndrome: a dissection of the genetic aetiology and candidate chromosome regions. J Med Genet 2001; 38: 81019. Quinn CT, Miller ST. Risk factors and prediction of outcomes in children and adolescents who have sickle cell anaemia. Hematol Oncol Clin North Am 2004; 18: 133954.

Saal HM. Russell-Silver syndrome. In: Cassidy SB, Allanson JE, eds. Management of genetic syndromes. New York: Wiley-Liss; 2005, p. 48996. Sontheimer RD. The management of dermatomyositis: current treatment options. Expert Opin Pharmacother 2004; 5: 108399.

PAEDIATRICS AND CHILD HEALTH 17:9

377

2007 Elsevier Ltd. All rights reserved.

SELF-ASSESSMENT

Self-assessment
Questions
Question 1 A baby boy was born at 35 weeks of gestational age to a primigravida mother. He had been antenatally diagnosed with hydrops fetalis with an absent corpus callosum. Antenatal karyotyping of pleural uid had revealed a chromosome 13q deletion. Postnatally, he was also detected to have a depressed nasal bridge and overlapping toes. He had delayed passage of meconium after birth and required rectal suppositories and rectal washouts to pass meconium. He had difculty in establishing enteral feeding due to persistent bilious aspirates and abdominal distension. At 2 weeks of age, he was still unable to tolerate enteral feeding, and further investigations were carried out. 1.  Which of the following additional investigations would you carry out in the initial postnatal period? a.  Cranial ultrasound, renal tract ultrasound, echocardiogram and ophthalmic examination b.  CT scan of the brain and skeletal survey c.  Renal tract ultrasound, CT scan of the brain and echocardiogram d.  Renal tract ultrasound, echocardiogram, CT scan of the brain and ophthalmic examination e.  Chest X-ray, skeletal survey, CT scan of the brain, renal tract ultrasound and echocardiogram 2.  What are the most relevant investigations for gastrointestinal diagnosis? a.  Abdominal ultrasound scan followed by CT scan of the abdomen b.  Upper and lower gastrointestinal contrast study followed by rectal biopsy c.  Abdominal ultrasound, CT scan of the abdomen and rectal biopsy d.  Abdominal X-ray and abdominal ultrasound e.  Abdominal X-ray and rectal biopsy 3.  Which of the following diagnoses is unlikely for gastrointestinal symptoms? a.  Pyloric stenosis b.  Hirshprungs disease c.  Malrotation d.  Gastrointestinal obstruction e.  Both a and b Question 2 A 20-week anomaly scan was performed on a 20-year-old woman and revealed exomphalos, macroglossia, bilateral enlarged kidneys and an umbilical cord cyst in the fetus. Amniocentesis conrmed a normal male karyotype. The baby was delivered at 38 weeks. He was found to be macrosomic, with a birthweight of 5.34 kg, and had macroglossia and transient hypoglycaemia. 1.  What is the most likely diagnosis? a.  Infant of a diabetic mother b.  BeckwithWiedemann syndrome c.  Perlman syndrome d.  SimpsonGolabiBehmel syndrome e.  PraderWilli syndrome 2.  What other associated features maybe noted in infancy? a.  Facial dysmorphism b.  Skeletal defects c.  Congenital cardiac defects d.  Hemi-hypertrophy e.  All of the above 3.  What long-term follow-up would you arrange for in an infant with this condition? a.  Renal ultrasound and serum alphafetoprotein level b.  Renal ultrasound and DMSA c.  Blood glucose level and regular growth monitoring d.  Echocardiogram and renal ultrasound e.  Echocardiogram and blood glucose Question 3 A male infant was born at term to a 44-year-old primigravida mother; he was in good condition and required no resuscitation. Echogenic bowel had been noted on an antenatal ultrasound scan. The infant was noted to have non-bilious vomiting on day 1 followed by abdominal distension and a palpable mass in the right iliac fossa by day 3. He had not passed meconium. Lower gastrointestinal contrast investigation (Figure 1) was performed on day 3, following which he passed meconium plugs. 1.  What is the most likely underlying condition? a.  Hirschsprungs disease b.  Malrotation c.  Duodenal atresia d.  Hypothyroidism e.  Cystic brosis 2.  What other investigation would you perform to conrm the underlying condition? a.  Rectal biopsy b.  CT scan of the abdomen

Suchita Joshi MRCPCH is Clinical Fellow in Neonatology at the University Hospital of Wales, Cardiff, UK. Jennifer Calvert BA (Oxon) BM BCh MRCP (UK) MRCPCH is Consultant Neonatologist at the University Hospital of Wales, Cardiff, UK.

PAEDIATRICS AND CHILD HEALTH 17:10

407

2007 Elsevier Ltd. All rights reserved.

SELF-ASSESSMENT

Answers
Answer 1 1.  a.  Cranial ultrasound, renal tract ultrasound, echocardiogram and ophthalmic examination. 2.  b.  Upper and lower gastrointestinal contrast study followed by rectal biopsy to exclude gastrointestinal obstruction and Hirshprungs disease respectively. 3.  a. Pyloric stenosis 13q deletion syndrome is known to be associated with gastrointestinal anomalies including Hirshprungs disease, duodenal atresia, intestinal malrotation, imperforate anus and small intestinal aplasia. Other associations include skeletal defects such as mandibular cleft, anomalies of the hands and feet and abnormal vertebrae; cardiac defects including atrial septal defect, ventricular septal defect, tetralogy of Fallot, patent ductus arteriosus, coarctation of the aorta, situs inversus and common aortopulmonary trunk. It is also associated with ophthalmic anomalies such as microphthalmos, hypertelorism, colobomata, retinoblastoma and aniridia. Neuromuscular associations include microcephaly, hypotonia, hypoplasia or aplasia of the corpus callosum and forebrain, anencephaly, hydrocephaly, aplasia of the olphactory tracts, optic nerve hypoplasia and encephalocele. Renal anomalies and malformed ears are also known to be associated. Recent studies have shown that deletions limited to proximal bands (q13q31) are marked mainly by growth retardation but no major malformations. Patients lacking the 13q32 band are the most seriously affected, and distal deletions are usually complicated by severe developmental delay. Hirshprungs disease, malrotation and gastrointestinal obstruction are all possible differential diagnosis in an infant with bilious aspirates and abdominal distension. Pyloric stenosis is unlikely to present in the rst 2 weeks. Answer 2 1.  b. BeckwithWiedemann syndrome 2.  d. Hemi-hypertrophy 3.  a. Renal ultrasound and serum alphafetoprotein level BeckwithWiedemann syndrome (BWS) is a multigenic disorder caused by dysregulation of the expression of imprinted genes in the 11p15 chromosomal region, most frequently of maternal origin, rearrangement of 11p15 as a result of paternal 11p duplication, or loss of normal imprinting.1 However, most cases are sporadic with a female predominance. This region has close associations to insulin-like growth factor and tumour-suppressor genes, and genes implicated in the regulation of insulin release. Approximately 80% of cases with features suggestive of BWS are known to be associated with an abnormality in chromosome 11. Differentiating BWS from other overgrowth syndromes such as Perlman and SimpsonGolabiBehmel syndromes can be

Figure 1

c.  Genetic testing for cystic brosis d.  Thyroid function test e.  Ultrasound of the thyroid gland Question 4 A fetus was found, on routine anomaly scanning at 20 weeks, to have a cystic lung lesion suggestive of a congenital cystic adenomatoid malformation of the lung (CCAM). 1.  Which of the following features is associated with poor prognosis in an infant with CCAM? a.  Hydrops b.  A bilateral lesion c.  The presence of other congenital anomalies d.  Both a and b e.  Both b and c 2.  What postnatal investigation should be carried out to conrm the diagnosis? a.  Chest X-ray b.  CT scan of the chest c.  MRI of the chest d.  Bronchoscopy and lung biopsy e.  Chest X-ray followed by MRI of the chest 3.  What long-term complications can arise in CCAM? a.  Pneumothorax and infection b.  Malignancy c.  Pleural effusion d.  Both a and b e.  Both a and c

PAEDIATRICS AND CHILD HEALTH 17:10

408

2007 Elsevier Ltd. All rights reserved.

SELF-ASSESSMENT

ifcult as many features may overlap. SimpsonGolabiBehmel d syndrome is also associated with high risk of cardiac arrhythmia and embryonic tumours in early childhood. An infant of a diabetic mother can present with macrosomia and neonatal hypoglycaemia but is unlikely to have abdominal wall defects or renal anomalies. PraderWilli syndrome is more likely to present with neonatal hypotonia and feeding difculty in infancy. Excessive weight gain is usually secondary to increased appetite later on in childhood. BWS is associated with abdominal wall defects, macroglossia, macrosomia, visceromegaly, hemi-hypertrophy, transverse creases on the ear lobes and hyperinsulinaemic hypoglycaemia. Facial dysmorphism, structural cardiac defects and skeletal defects are not known associations. BWS is associated with an increased risk of embryonic tumours such as Wilms tumour, hepatoblastoma and adrenal carcinoma, and this risk is increased in the presence of hemihypertrophy. The overall risk of Wilms tumour in those with 11p15 chromosomal abnormality is 510%. Is has been recommended that Wilms tumour surveillance should be offered to children with hemi-hypertrophy with paternal uniparental disomy of 11p15. Surveillance should be carried out by renal ultrasonography every 34 months and no less frequently than three times a year until the age of 7 years.2,3 Children with BWS and Wilms tumour have excellent prognosis with modern treatment regimens. There is also an increased risk of hepatoblastoma in BWS in the rst 4 years of life. Serum alphafetoprotein level is almost always elevated in hepatoblastoma and is a sensitive marker of the disease. It has therefore been suggested that these children are also screened with 3-monthly alphafetoprotein levels until the age of 4 years.3 Answer 3 1.  e. Cystic brosis 2.  d. Genetic testing for cystic brosis The incidence of cystic brosis (CF) in the UK is 1 per 2000 children born. Meconium ileus causes intestinal obstruction in up to 15% of newborn children with CF. The condition may be detected antenatally by the presence of a bright or echogenic bowel due to the presence of inspissated meconium. After birth, infants may present with failure to pass meconium, abdominal distension and bile-stained vomiting. Contrast enema typically demonstrates the presence of a microcolon. It may also soften the meconium mass to relieve obstruction. However, the majority of cases require surgical intervention to remove impacted meconium and to remove any non-viable gut. Rarely, antenatal meconium peritonitis, neonatal volvulus and atresias may occur. CF is an autosomal recessive condition. The most common mutation in the Caucasian population is deltaF508. If genetic testing in the neonatal period is not conclusive and there is a high index of suspicion, the infant may require a sweat test, which can usually be carried out from 6 weeks of age.

Answer 4 1.  d. Both a and b 2.  b. CT scan of the chest 3.  d. Both a and b CCAM is a congenital anomaly of the fetal lung thought to result from an arrest in lung development. Both sexes are affected equally. Approximately 3% of cases occur bilaterally. The middle or upper lobes are usually affected. Antenatally, a small number of CCAMs can be associated with the development of hydrops or polyhydramnios, and antenatal interventions in the form of thoraco-amniotic shunts and percutaneous intrauterine laser therapy have been successfully reported. However, the majority of cases have a good prognosis and most babies are asymptomatic at birth. A poor prognosis is associated with bilateral lesions or the development of hydrops.4 About half the cases of CCAM detected in early pregnancy show some regression antenatally, although the vast majority of cases persist postnatally. Chest X-ray has been shown to be poor at detecting CCAM and can appear normal in the newborn period; however the majority of these lesions are demonstrable postnatally on CT scanning, which is highly sensitive and is therefore the recommended mode of imaging for following up babies diagnosed antenatally with this condition. The presence of abnormal lung tissue can predispose to infection, and pneumothoraces have been reported. In addition, there are now many case reports of bronchoalveolar carcinoma and pleuropulmonary blastoma occurring in CCAM. Because of these potential long-term complications, elective surgery has been recommended within the rst year of life for persistent, asymptomatic cases of CCAM.5 Cases that are symptomatic may require earlier intervention.

REFERENCES 1 Eden T, Makin G. Oncology. In: McIntosh N, Helms P, SmythR, eds. Forfar and Arneils textbook of Paediatrics, 6th edn. Churchill Livingstone, 2003, p. 104899. 2 Scott RH, Walker L, Olsen OE, et al. Surveillance for Wilms tumour in at risk children: pragmatic recommendations for best practice. Arch Dis Child 2006; 91(12): 9959. 3 Tan TY, Amor DJ. Tumour surveillance in BWS and hemihyperplasia: a critical review of the literature and suggested guidelines for local practice. J Paediatr Child Health 2006; 42(9): 48690. 4 Bunduki V, et al. Prognostic factors associated with congenital cystic adenomatoid malformation of the lung. Prenat Diagn 20: 45964. 5 Calvert JK, Lakhoo K. Antenatally suspected congenital cystic adenomatoid malformation of the lung: postnatal investigation and timing of surgery. J Pediatr Surg 2007; 42(2): 41114.

PAEDIATRICS AND CHILD HEALTH 17:10

409

2007 Elsevier Ltd. All rights reserved.

SELF-ASSESSMENT

Self-assessment
Questions
Question 1 You are called to the neonatal intensive care unit by one of the nurses who is concerned about swelling and redness at an intravenous drip site. The patient has been receiving an intravenous infusion of an electrolyte solution containing potassium. When you arrive and assess the patient, all vital signs are stable but the childs arm is swollen and red. (a)  Which rst aid treatment method would be inappropriate? Topical application of ice  Injection of small amounts of saline to dilute the toxic agent Injection of hyaluronidase Injection of alcohol Local injection of steroid to combat inammation  Injection of specic antidotes, such as phentolamine in the case of extravasation of vasopressor agents (b)  Intravenous infusion uids that are dangerous if extravasated are termed vesicants. Which uid is NOT a vesicant? Vinblastine Aminophyllines Diazepam Doxorubicin 0.9% Saline Concentrated parenteral nutrition (c)  Which one of the following statements is true?  Extravasation injuries do not need to be referred for plastic surgery specialist assessment  Neonates and premature babies are particularly at risk of extravasation injury because of the small size of their vessels and the immature structure of their skin  Multiple venepuncture sites in the same area can help prevent extravasation  Placing small cannulas in small veins is the best way to safeguard against extravasation. Question 2 For each of the following clinical descriptions, pick the term from the list below that describes the congenital hand condition: polydactyly Syndactyly camptodactyly macrodactyly clinodactyly. (a)  Localised enlargement of all structures within a nger (b)  Lateral deformity of the nger (c)  More than ve ngers present in a hand Question 3 You are looking after a child on the ward who was born with a cleft palate. His parents want to conceive again and ask you about the causes of cleft palate. They want to know what the chances are that another baby born to them will have a cleft lip or palate. (a)  Which one of the following factors is NOT a causative factor for congenital cleft palate? Maternal consumption of alcohol Maternal use of anticonvulsants Maternal folate supplements Maternal diabetes Maternal use of retinoic acid (b)  Which of the following statements is true and which false?  Isolated cleft palate is associated with environmental factors rather than familial factors whereas cleft lip or cleft lip combined with cleft palate has a familial association  Cleft palate is associated with congenital syndromes but cleft lip never is  Cleft palate is twice as common in female babies but cleft lip or cleft lip with cleft palate is twice as common in male babies  Embryological palatal development occurs between the 7th and 10th weeks of intra-uterine life. The lip is formed earlier between the 4th and 8th weeks. Question 4 (a)  Which one of the following is NOT associated with hypospadias? Inguinal hernias Undescended testes A urethra opening onto the dorsal surface of the penis A ventral curvature of the penis (chordee) Urethral valves Paraurethral sinuses A attened glans penis A hooded prepuce Question 5 You are the paediatric specialist registrar on call and are called by the plastic surgery registrar who asks you to review a 2- year-old boy brought in by his mother and grandmother with a scald burn. The history given is that the burn occurred 2 days ago when the child climbed into a hot bath, his mother heard him scream and found him in the bath. She then performed rst aid by running the childs legs under cold water and the burns seemed to improve so did not summon medical

Christopher James BSc BM BCh MRCPCH is Paediatric Specialist Registrar at the Bristol Childrens Hospital, Bristol, UK. Jennifer ONeill BM BCh MA MRCS MMed is Plastic Surgery Specialist Registrar at Frenchay Hospital, Bristol, UK.

PAEDIATRICS AND CHILD HEALTH 17:11

460

2007 Elsevier Ltd. All rights reserved.

SELF-ASSESSMENT

help at that time. The scalds are in a sock-like distribution with a clear-cut immersion line and there are no splash marks. There are no other bruises or injuries seen. The plastic surgery team tell you that they are concerned about the possibility of non-accidental injury and ask for your opinion. (a)  What are the indicators of possible non-accidental injury? (Choose two) Unexplained delay in seeking medical attention  The patient is brought into hospital by his or her parents Scalds with no splash marks No other bruises or welts or fractures  History of injury consistent with the developmental capacity of the child (b)  Which one of the following actions should you NOT take when you go to review this patient: Take the history again to see if it is consistent  Check to see if the child is on the child protection register  Look through past medical records of the patient and siblings to see if there is previous history of injury or accident  Admit the patient to hospital for safety while you investigate  Challenge the parent as to what happened and ask them to explain why they are abusing their child

Syndactyly is a condition where two or more digits are fused together (Figures 1 a and b) and camptodactyly is a congenital exion deformity at the proximal interphalangeal joint. Question 3 (a)  Maternal folate supplements  All are thought to be causative factors of cleft palate except for maternal folate supplements there is some evidence that maternal folate supplements may reduce the incidence of cleft palate. (b)  The key to understanding the causative factors of cleft lip and palate is to distinguish between isolated cleft palate and cleft lip that may be combined with cleft palate or not.  Isolated cleft palate is associated with environmental factors rather than familial factors whereas cleft lip or cleft lip combined with cleft palate has a familial association TRUE  The relative risk of a child having cleft lip or cleft lip and palate if one sibling is affected is approximately 4% compared with 0.1% if there is no history of cleft lip or cleft lip and palate in the family.  Cleft palate is associated with congenital syndromes but cleft lip never is FALSE  Cleft lip or cleft lip combined with cleft palate is infrequently associated with other abnormalities or syndromes whereas cleft palate is more likely to be associated with these conditions. Cleft palate is associated with the following syndromes: velocardiofacial syndrome (VCF), Treacher Collins syndrome, Sticklers syndrome, Aperts syndrome, Crouzons syndrome, Downs syndrome, Pierre Robin Sequence. Cleft lip, although not usually associated with a syndrome may be associated with a few syndromes including Van der Woude syndrome a syndrome with autosomal dominance, multiple pits in the lips and absence of second premolar teeth.  Cleft palate is twice as common in female babies but cleft lip or cleft lip with cleft palate is twice as common in male babies TRUE  Embryological palatal development occurs between the 7th and 10th weeks of intra-uterine life. The lip is formed earlier between the 4th and 8th weeks TRUE Question 4 (a)  A urethra opening onto the dorsal surface of the penis  The above is termed epispadias. Hypospadias is an abnormally proximal position of the urethral meatus on the ventral aspect of the penis or scrotum. (NB the penis is erect in the anatomical position so the ventral surface is the underside.) Question 5

Answers
Question 1 (a)  Injection of alcohol alcohol is a vesicant itself (b)  0.9% Saline (c)  Neonates and premature babies are particularly at risk of extravasation injury because of the small size of their vessels and the immature structure of their skin The following statements were false:  Extravasation injuries do not need to be referred for plastic surgery specialist assessment most extravasation injuries need referral and assessment by the plastic surgeon at the earliest opportunity to see if early surgical debridement and grafting is indicated.  Multiple venepuncture sites in the same area can help prevent extravasation multiple venepuncture sites create leaky veins and may be a cause of extravasation.  Placing small cannulas in small veins is the best way to safeguard against extravasation although small cannulas limit the amount of extravasation, the best vein to prevent extravasation is a large intact vein with good blood ow. Question 2 (a)  Macrodactyly (b)  Clinodactyly (c)  Polydactyly

(a)  Unexplained delay in seeking medical attention  Scalds with no splash marks

PAEDIATRICS AND CHILD HEALTH 17:11

461

2007 Elsevier Ltd. All rights reserved.

SELF-ASSESSMENT

Figure 1 A case of Syndactyly.

 All of the above are indicators of possible non-accidental injury. The scenario given has these indicators within it. The scalds without splash marks indicate that the child has been dipped or held in hot water rather than accidentally being exposed to it. If the child is brought in by an unrelated adult, rather than the parents, this can also be an indicator as can other injuries such as bruises, welts or fractures. (a)  Challenge the parent as to what happened and ask them to explain why they are abusing their child.

 It is not appropriate for you to challenge or accuse the parent in any way. All other actions are considered appropriate.

FURTHER REaDING Richards AM. Key notes on plastic surgery. Oxford: Blackwell Publishing; 2002. www.core-info.cf.ac.uk

PAEDIATRICS AND CHILD HEALTH 17:11

462

2007 Elsevier Ltd. All rights reserved.

SELF-ASSESSMENT

Self-assessment
Questions
Question 1 A 4-month-old girl was brought to hospital because of being generally unwell with complaints of poor feeding, mild vomiting and two to three episodes of loose stools over the previous day. On examination she was alert and was estimated to be about 5% dehydrated, and still passing urine. Her admission weight was 4.9 kg. The results of her initial investigations were: Serum sodium 189 mmol/l Serum potassium 3.1 mmol/l Urea 11.9 mmol/l Creatinine 49 mol/l Urine sodium 297 mmol/l Urine osmolality 879 mosm/l Urine creatinine 2 mmol/l (a)  What is the most likely diagnosis? Nephrogenic diabetes insipidus (DI) Central DI Salt poisoning Reset osmostat Conns syndrome (b)  Which further investigation is least important? Feed analysis Growth hormone assessment Weight after rehydration Antidiuretic hormone (ADH) concentration Renal ultrasound (c)  What would be your immediate management plan? Intranasal DDAVP (desmopressin acetate) Immediate reduction of serum sodium by 5 mmol/l with 5% dextrose bolus Peritoneal dialysis Continue bottle feeding Rehydration and gradual reduction of serum sodium Question 2 A 15-year-old boy with end stage renal failure secondary to renal dysplasia on haemodialysis presented with knee pain. A plan x-ray (Figure 1) of the knee joint was performed; the results of blood investigations are as mentioned below. Calcium Phosphate 2.14 mmol/l 2.11 mmol/l

Figure 1 Plan x-ray of the knee joint.

Alkaline phosphatase Parathyroid hormone (PTH) 25 hydroxy vitamin D

403 IU/l (100400 IU/l) 73 pmol/l (0.95.4) 31 ng/ml (850)

(a)  What is the most likely cause of the abnormalities on this x-ray? Brown tumour secondary to renal osteodystrophy Congenital bone cysts Osteosarcoma of the distal femur Osteoporosis secondary to hypercalciuria Osteomalacia (b)  How do you best explain these abnormalities? Cholecalciferol deciency secondary to poor diet Hypercalciuria, causing hyperparathyroidism A high phosphate diet Secondary hyperparathyroidism Vitamin D dependent rickets (c)  Which treatment would you not consider in treating this boy? Phosphate removal by intensifying dialysis A low phosphate diet A high calcium diet Calcium carbonate for phosphate binding 1 alfa calcidol Question 3 A 15-year-old girl with known cerebral palsy had orthopaedic surgery for a dislocated hip. Five days following the procedure

Hemanth H Jakka MBBS MRCPCH-I & II is Senior House Ofcer in Paediatric Nephrology at the University Hospital of Wales, Cardiff, UK. Judith Van der Voort Artsexamen (NL) MRCP MRCPCH is Consultant Paediatric Nephrologist, University Hospital of Wales, Cardiff, UK.

PAEDIATRICS AND CHILD HEALTH 17:12

492

2007 Elsevier Ltd. All rights reserved.

SELF-ASSESSMENT

she became unwell with loose stool followed by a high fever, tachycardia, hypotension and shock. She was resuscitated with uid bolusses and was started on antibiotics, but was found to be passing very little urine. Results of her investigations performed were: Sodium 145 mmol/l Potassium 5.1 mmol/l Urea 32.5 mmol/l Creatinine 290 mol/l Calcium 1.76 mmol/l Phosphate 3.5 mmol/l Haemoglobin 9.5 g/dl White cell count 9.6 109/l Platelets 221 109/l Albumin 28 g/l Alkaline phosphatase 78 IU/l (70250) Aspartate aminotransferase (AST) 4964 IU/l (545) Alanine aminotransferase (ALT) 161 IU/l (540) Bilirubin 4 mol/l (122) Creatinine kinase (CK) 418,700 IU/l (20175) (a)  What is the most likely cause of her acutely impaired renal function? Pre-renal failure Acute tubular necrosis Rhabdomyolysis Acute hepatic failure Haemolytic uraemic syndrome (b)  On urine dipstick 4+ blood was found, but on microscopy no red cells were seen. What is the most likely cause for this? Haemoglobinuria Myoglobinuria Acute tubular necrosis Acute glomerulonephritis Acute haemolysis (c)  Which possible further management strategy is least appropriate in this case? Fluid resuscitation if patient is still hypovolaemic Haemodialysis Peritoneal dialysis Plasmapheresis Alkalinisation of urine if urine output is maintained

must be identied promptly as this helps in the appropriate management. The main causes are either due to water deciency or sodium excess. Water deciency may be due to an inadequate intake (e.g. poor breast feeding) or abnormal losses (e.g. central and nephrogenic DI, increased insensible losses) or a combination of the two. In children loss of hypotonic uids (e.g. gastroenteritis) is a common aetiology for hypernatraemic dehydration. A mentally competent healthy child will not become hypernatraemic from water deciency alone provided there is free access to water, as even mild hypertonicity is a powerful stimulus to thirst. The other important cause to consider is salt overload, which can either be accidental or can be non-accidental (salt poisoning). Patients with reset osmostat have a reduced threshold for ADH release and as a result the plasma sodium is below normal but stable. After clinical assessment and assessment of hydration status, including weight measurement, urine must be sent for sodium content and osmolality. High sodium excretion can be established by measuring the urine sodium concentration (less than 20 mmol/l considered sodium preservation, and more than 40 mmol/l considered sodium excretion) or more accurately with the calculation of fractional excretion of sodium:
FE Na (%)=(U Na PCr ) / (U Cr PNa ) 100.

(Subscripts Na and Cr denote sodium and creatinine values in P = plasma and U = urine.) FENa is generally <1% in situations of water loss and rises if either the glomerular ltration rate (GFR) is reduced or if an increased sodium load is being excreted. Hypernatremia and a FENa >1%, with normal renal function is mostly suggestive of salt overload. Low urine sodium excretion, suggestive of salt preservation can be due to reduced intravascular volume secondary to extrarenal losses (e.g. increased gastrointestinal or insensible losses) in which case the urine osmolality will be high (generally above 600 mosm/kg). If salt preservation is associated with renal uid losses, urine osmolality will be low (less than 300 mosm/kg), although urine osmolality might be higher when urine losses are due to partial forms of DI. The aim of treatment, after initial stabilisation and restoration of adequate hydration status is to gradually lower the plasma sodium concentration by not more than 1012 mmol/l per 24 hours. Hypernatraemia can cause cellular dehydration, particularly within the brain cells and a rapid drop of intravascular osmolality can cause a uid shift into cells predisposing to cerebral oedema.

Answers
Question 1 (a)  Salt poisoning (b)  Growth hormone assessment (c)  Rehydration and gradual reduction of serum sodium Hypernatraemia and salt poisoning in children Hypernatraemia is dened as a serum sodium concentration greater than 145 mmol/l. The underlying mechanism

Features to help differentiate between hypertonic dehydration and salt overload


Hypertonic dehydration History of dehydration Signs of dehydration Body weight Urea and creatinine Fractional sodium excretion Present May be present (weight loss) elevated <1% Salt overload

Absent/not signicant Absent/not signicant Normal or Normal >1%

PAEDIATRICS AND CHILD HEALTH 17:12

493

2007 Elsevier Ltd. All rights reserved.

SELF-ASSESSMENT

FURThER REaDING Coulthard MG, Haycock GB. Distinguishing between salt poisoning and hypernatraemic dehydration in children. BMJ 2003; 326: 15760. Haycock GB. Hypernatraemia: diagnosis and management. Arch Dis Child Ed Pract 2006; 91: ep8. Meadow R. Non-accidental salt poisoning. Arch Dis Child 1993; 68: 44852.

FURThER REaDING Adams JE. Renal bone disease: Radiological investigation. Kid Int 1999; 56 (suppl 73): S38S41. Klaus G, Watson A, Edefonti A, et al. for the European Dialysis Working Group. Prevention and treatment of renal osteodystrophy in children on chronic renal failure: European guidelines. Pediatr Nephrol 2006; 21: 1519.

Question 3 Question 2 (a)  Brown tumour secondary to renal osteodystrophy (b)  Secondary hyperparathyroidism (c)  A high calcium diet Renal bone disease renal osteodystrophy A brown tumour is a cavity in bone where there is excessive osteoclastic resorption, brous tissue necrosis and liquefaction. Such cavities appear as cysts in the bone, can occur anywhere in the skeleton and may expand bone. These cystic lesions are due to renal osteodystrophy, a high turnover bone disease secondary to renal failure. A congenital bone cyst would have a similar appearance on x-ray, but is unlikely in this case given the background history of chronic renal failure. The radiological abnormalities of osteoporosis and osteomalacia include fractures and reduced bone mineral density, but not cystic lesions. The low calcium concentration in renal failure is caused by reduced hydroxylation of 25 hydroxycholecalciferol to 1,25 dihydroxycholecalciferol, which takes place in the kidney. This is often combined with hyperphosphatemia, caused by phosphate retention. Phosphate is cleared by the kidney, but once the GFR drops signicantly the kidney is unable to clear phosphate sufciently and it will be retained. The low calcium, high phosphate and low activated vitamin D concentration will increase PTH production. PTH will increase calcium and phosphate resorption from bone, further increasing phosphate concentration. With normal renal function the hyperparathyroidism will cause phosphaturia, but because of the low GFR, the kidney cannot remove the phosphate load. Although this would be made worse by a diet high in phosphate, this is not the primary cause. In nutritional rickets a drop in phosphate will occur because of the phosphaturic action of PTH and a low 25 hydroxycholecalciferol concentration will be found, because of low vitamin D substrate intake. Hypercalciuria does not cause hyperparathyroidism, unless it leads to hypocalcaemia. The treatment for high turnover renal osteodystrophy in renal failure is a combination of removing phosphate through dialysis and phosphate binders and occasionally a low phosphate diet. Alfacalcidol, the activated from of vitamin D will suppress PTH production and improve calcium absorption from the gut. (a)  Rhabdomyolysis (b)  Myoglobinuria (c)  Plasmapheresis Rhabdomyolysis and acute renal failure Rhabdomyolysis is a systemic metabolic disorder caused by leakage of muscle cell constituents into the blood following necrosis of skeletal muscle. Rhabdomyolysis is a chronic ongoing process in children with congenital muscle disease, but can be acute in children due to direct trauma to the muscle, muscle necrosis from ischaemia, inammation or exposure to drugs and toxins. In the above described case the most likely cause for the severe rhabdomyolysis is a combination of muscle trauma from the orthopaediac surgery, ischaemia from the positioning and dehydration post-operatively. Dehydration in combination with aciduria is known to make myoglobinaemia more toxic leading to acute renal failure. Clinical ndings include fever, muscle pain, hypotension secondary to uid third spacing, with acute renal failure and oliguria. Laboratory ndings may include acute renal failure with elevated urea and creatinine, a high AST, without liver enzyme abnormalities, a very high CK and lactate dehydrogenase. Urine dipstick will show haematuria due to myoglobinuria, with no red cells found on microscopy. Some proteinuria is usually present. Management includes removing the causative factor if possible, early vigorous intravenous uid therapy, often administered with diuretics and sodium bicarbonate. Dialysis will be necessary if renal output is not re-established after uid therapy but this is purely symptomatic as myoglobin is poorly removed by either peritoneal or haemodialysis.

FURThER REaDING Watanabe T. Rhabdomyolysis and acute renal failure in children. Pediatr Nephrol 2001; 16: 10725. Zager RA. Rhabdomyolysis and myohemoglobinuric acute renal failure. Kidney Int 1996; 49: 31426.

PAEDIATRICS AND CHILD HEALTH 17:12

494

2007 Elsevier Ltd. All rights reserved.

Você também pode gostar